Saturday, December 15, 2007

80 - COMEDK 2007 - 1 - 180 bits

1-The preganglionic parasympathetic fibers to the parotid gland travel in

a. Lesser petrosal nerve
b. Great petrosal nerve
c. Deep petrosal nerve
d. Internal carotid nerve

----------------------------------------------

The correct answer is A

The innervation of the parotid involves the preganglionic parasympathetic portion of the gland, beginning in the inferior salivatory nucleus and passing via the glossopharyngeal nerve, the tympanic plexus, and the lesser petrosal nerve to the oticganglion.

The postganglionic fibers travel along the auriculotemporal nerve and into the parotid gland, where they control glandular secretions.

Preganglionic sympathetic fibers originate in the first and second thoracic segments and synapse in the cervical sympathetic ganglia. Postganglionic sympathetic fibers travel along the external carotid artery to the superficial temporal artery and the transverse facial artery and terminate in the sweat glands.

------------------------------------------

2-Hind milk is richer in

a. Carbohydrate
b. protine
c. Fat
d. Minerals

----------------------------------------------

The correct answer is C

Hind-milk has the highest concentration of fat and is released after several minutes of nursing. It is similar in consistency to cream and will have a soporific (sleepy) effect on your baby. Hind-milk is important for your baby to feel satisfied and to gain adequate weight.

------------------------------------------------

3-The test of choice to detect perivalvular abcess of the aortic valve is

a. MRI of heart
b. Transesophageal Echo with doppler
c. Ventriculography
d. CT Chest

---------------------------------------------------

The correct answer is B

Perivalvular abscesses are seen in about 30% of patients with bacterial endocarditis. It is important to detect these lesions, because they indicate advanced disease that has a poor prognosis and thus necessitates aggressive medical management and sometimes surgery. Transesophageal echocardiography (TEE) is the method of choice for detecting infective endocarditis of both native and prosthetic cardiac valves. This technique also aids in the detection of complications, such as perivalvular abscesses and pseudoaneurysms, with good sensitivity (80%–87%) and specificity (95%–98%)
------------------------------------------------

4-Adequate recovery of neuromuscular function after reversal is indicated by

a. Minute volume
b. Respiratory rate
c. Tidal volume
d. Headlift for 3 seconds

----------------------------------------------------

The correct answer is D

When used as an antagonist to nondepolarizing muscle relaxants, adequate recovery of voluntary respiration and neuromuscular transmission must be obtained prior to discontinuation of respiratory assistance, and there should be continuous patient observation. Satisfactory recovery may be judged by adequacy of skeletal muscle tone, respiratory measurements, and by observation of the response to peripheral nerve stimulation. A patent airway should be maintained and manual or mechanical ventilation should be continued until complete recovery of normal respiration is assured.

Head-lift for 5 s has been used clinically for the assessment of residual block.

-------------------------------------------------------

5-The commonest site of lymphoma in the gastrointestinal system is

a. Small bowel
b. Stomach
c. Large intestine
d. Esophagus

-----------------------------------------------------------

The correct answer is B


Gastrointestinal lymphomas typically present with nonspecific signs and symptoms attributable to the site of involvement.

Primary gastric lymphoma accounts for 3 percent of gastric neoplasms and 10 percent of lymphomas. The stomach is the most common extranodal site of lymphoma and is also the most common site of gastrointestinal lymphoma

* Stomach — 75 percent
* Small bowel (including duodenum) — 9 percent
* Ileo-cecal region — 7 percent
* More than one GI site — 6 percent
* Rectum — 2 percent
* Diffuse colonic involvement — 1 percent

--------------------------------------------------------------------

6-The form of vascular disease responsible for malignant hypertension is

a. hyaline arteriolosclerosis
b. Arteriosclerosis obliterans
c. Hyperplastic Arteriolosclerosis
d. Medial calcific sclerosis

-------------------------------------------------------------------------

The correct answer is C

Hyperplastic arteriolosclerosis is commonly associated with malignant HTN. With ongoing injury, the vascular walls hypertrophy due to hyperplasia of SMCs and sometimes this occurs along with necrosis of the vessel wall. There is a characterisitic “onion-skin” appearance of the vessel wall.


Hyaline Arteriolosclerosis: The lesions are characterized by glassy thickening of arterial and arteriolar walls. Hyaline arteriolosclerosis is seen in elderly patients, with or without hypertension or diabetes, and in patients with long-standing diabetes but the lesions are most common and most severe in hypertensive patients.

------------------------------------------------------------------

7-Photopsiae occurs in

a. Iritis
b. Cyclitis
c. Choroiditis
d. Retinitis

----------------------------------------------------------------------

The correct answer is D

Photopsia is the presence of perceived flashes of light. It is most commonly associated with posterior vitreous detachment, migraine with aura, migraine aura without headache and retinal break or detachment.
-------------------------------------------------------

8-A 2 yr old child with a head circumference of 55 cms is likely to have had intrauterine infection due to

a. Rubella
b. Toxoplasmosis
c. Herpes
d. Hepatitis B

-----------------------------------------------------------

The correct answer is B


At birth a child has an average head circumference of around 35 cms. This increases at the rate of 1.4 cms per month for first four months.

This child has macrocephaly...

Microcephaly may occur in neonates with central nervous system toxoplasmosis infection but macrocephaly due to hydrocephalus as a result of aqueductal stenosis may also occur.

----------------------------------------------------------------

9-Stomach derives its blood supply from all these arteries directly or indirectly EXCEPT:

a. Splenic artery
b. Hepatic artery
c. Superior mesenteric artery
d. Coeliac axis

-----------------------------------------------------------

The correct answer is C

Blood supply of Stomach

* left gastric artery from celiac trunk
* splenic artery and left gastroepiploic artery
* common hepatic artery through the right gastroepiploic artery and gastroduodenal artery

-----------------------------------------------------

10- Bell's palsy is paralysis of

a. UMN V nerve
b. UMN VII nerve
c. LMN V Nerve
d. LMN VII nerve
-----------------------------------------------------

The correct answer is D


LMN paralysis of CN VII


* Bell's Palsy is a LMN paralysis in which there is an abolition of both voluntary and reflex movements of the facial muscles. On the ipsilateral side the face is masklike and the forehead immobile

* corneal sensitivity remains (CN V) but the patient is unable to blink or close the eyelid.

* lacrimation on the lesion side may be impaired (greater petrosal nerve)

* there may be a reduction of saliva (chorda tympani nerve)

* taste is lost on the ipsilateral anterior 2/3 of the tongue (chorda tympani)

* ear pain

* there can be an increased acuity to sound (hyperacusis) due to paralysis of the stapedius muscle

* when a lesion occurs at the location where VII emerges from the pons, as the patient opens his mouth, his eyes close (jaw-winking phenomenon)
---------------------------------------------------------------------------------------

11- The center of ossification of which of the following is used as medico legal evidence for fetal viability?

a. Head of femur
b. Distal end of femur
c. Greater trochanter
d. Lesser trochanter

------------------------------------------------------------------

The correct answer is B


Assessment of of age if Fetus is important in Cases of (Medico-legal Aspect) Criminal Abortion, Infanticide, Still-Born or Dead Born Baby, Fabricated Abortion, Marriage, Divorce, Adoption, Inheritance etc.

At the end of Ninth Month

*Length is about 45 cm
*Weight is about 2.5 Kg to 3 Kg
*Scalp hair is dark and 4cms long
*Meconeum is seen at the end of large intestine
*Ossification center are usually present in the lower end of femur, in Cuboid and Capitate bone

----------------------------------------------------------------------

12-All of the following are inhibitors of Cytochrome oxidase except

a. Carbon monoxide
b. Amytal
c. Cyanide
d. Azide

-------------------------------

The correct answer is B

Cytochrome oxidase (complex IV) carries out the following irreversible reaction:
O2 + 4 H+ + 4 e- ---> 2 H2O

Cyanide Blocks cytochrome oxidase (complex 4) and prevents both coupled and uncoupled respiration with all substrates, including NADH, succinate and ascorbate + TMPD.

----------------------------------------------

13-von willebrand factor is produced by

a. liver
b. platelets
c. endothelial cells
d. Spleen

------------------------------------------

The correct answer is C

Von Willebrand Factor (vWF) is a heterogeneous multimeric glycoprotein which is produced by endothelial cells and megakaryocytes. The molecular weight varies from 400 kD to 20,000 kD. In platelets, vWF is stored in the alpha-granules. In endothelial cells vWF is stored in so-called Weibel-Palade bodies. The normal function of vWF is to stabilize factor VIII, as well as to be a carrier protein of factor VIII. Large vWF multimers also stabilise platelet adhesion to the subendothelial matrix in case of tissue injury.

----------------------------------------------------

14-The definitive diagnosis of Pneumocystis Carinii Pneumonia is

a. Serological tests
b. Chest X-ray
c. Finding cysts in tissue specimen
d. CT scan of thorax

------------------------------------------------

The correct answer is A

PNEUMOCYSTIS

Pathogenesis:

* P. carinii is a low virulence fungus commonly found in lungs in humans and other mammals.
* Infects host via inhalation of the organism in its trophic form.
* Usually infection occurs in those HIV infected patients with CD4+ counts below 200/mm3, since CD4 + lymphocyte activation of macrophages is critical in host defense.
* Fungus cannot be targeted with usual antifungal agents.


Clinical characteristics:

* Shortness of breath, fever and nonproductive cough.
* Tachypnea and tachycardia characterize an acute ill patient. Alar flaring, intercostal reaction with radiographic findings of bilateral diffuse infiltrates are present.
* The most common finding is hypoxia.
* Extrapulmonary pheumocystosis may occur in cases of advanced HIV infection with involvement of liver, spleen, lymph nodes, GI tract, eyes, thyroid and adrenals.

Diagnostic work-up:

* From identification of organism in respiratory secretions or tissue sections of severely immunocompromised individuals with respiratory symptoms.
* Immunofluerescence is the most widely used technique, while demonstration of P. carinii antigen in serological test and DNA in tissue specimens are of high specificity and sensitivity.

Treatment:

* Key is early diagnosis. Mechanical ventilation along with Trimethoprin-sulfamethoxazole may reduce mortality from 80% -90% to 50%.
* TMP-SMX inhibits folic acid synthesis is relatively inexpensiveavailable in oral and parenteral forms is well tolerated and the drug of choice. Usual IV doses 15-20 mg/kg/day.
* Alternative agents Dapsone 100mg/day PO alone or in combonation with TMP-SMX.
* In patients with AIDS and CD4+ count below 200/mm3, oral candidiasis, unexplained fever of >100 F for more than 2 weeks, or recovered from previous episode of P. carinii pneunitis chemoprophylaxis is recommended and continued for life.

-----------------------------------------------------------

15-Myositis ossificans is

a. worm calcification
b. Callus formation
c. Regenerate
d. Post traumatic ossification

------------------------------------------------------

The correct answer is D


Myositis ossificans is a benign, ossifying soft-tissue lesion typically occurring within skeletal muscle. Patients are usually adolescents and young adults; myositis ossificans is rare in children under 10 years of age. The most frequent symptoms and signs are pain and tenderness with a soft tissue mass. Approximately 80% of cases arise in the large muscles of the extremities.

Myositis ossificans traumatica (latinized name) is produced as an insult (crushing injury) to muscle against bone. A hematoma (bloody mass) is formed against the periosteum and subsequently calcification and ossification occurs.

-------------------------------------------------------

16-Depressive delusions that the world and everything related to it cease to exist is called

a. Persecutory delusion
b. Delusion of infidelity
c. Nihilistic delusion
d. Delusion of reference

-----------------------------------------------------------

The correct answer is C

The depersonalisation in severe depression culminates in the so-called nihilistic delusions or the Cotard-Syndrome, formerly aptly called „melancholia anaesthetica“. The patients do not sense their body any more; taste, smell, even the sense of warmth or pain are gone, everything seems dead. This makes them conclude that they have already died and ought to be buried. They may even deny their own existence or the existence of the world.

---------------------------------------------------------------

17- Which vessel is most commonly associated with posterior Duodenal ulcer?

a. Right gastroepiploic artery
b. Common hepatic artery
c. Gastroduodenal artery
d. Superior mesenteric artery

-----------------------------------------------------------

The correct answer is C

Duodenal Ulcer

1. characteristics: mostly occurs in 40-65 yo, M:F = 2:1, more than twice as common as gastric ulcers, bleed 4x as often as gastric ulcers, 25% of total UGI bleeds
2. etiology
1. most (95%) due to H. Pylori
2. increased gastric acid production
3. environmental factors – smoking, NSAIDs
3. risk factors: male, smoking, aspirin/NSAID use, uremia, Zollinger-Ellison syndrome, H. pylori infection, trauma, burn injury

4. sx: epigastric pain (burning or aching, several hrs after meal, initially relieved by food, milk, or antacids), back pain, N/V, anorexia, decreased appetite
5. signs: tenderness in epigastric area (possibly), guaiac-positive stool, melena, hematochezia, hematemesis
6. ddx: acute abdomen, pancreatitis, cholecystitis, all causes of UGI bleeding, Zollinger-Ellison syndrome, gastritis, MI, gastric ulcer
7. diagnose with H&P, EGD, UGI series (if not actively bleeding), urea breath test, H. pylori serology

a. with EGD, visible vessel in ulcer crater a/w 90% rebleed rate

8. location

1. majority within 2 cm of pylorus in duodenal bulb
2. posterior – more likely to bleed b/c erodes into gastroduodenal artery
3. anterior – most likely to perforate (see free air under diaphragm on upright AXR)

9. medical tx
1. Antacids (Mg, Al, Ca): control gastric pH, promote healing
2. H2 receptor blockers: usually heal ulcers in 4-6 wks
3. Sucralfate: coats ulcer
4. Proton pump inhibitors (omeprazole): decrease acid production; used for refractory ulcers and ulcers a/w Zollinger-Ellison syndrome
5. Prostaglandins: provide mucosal protection
6. Bismuth
7. Antibiotics: triple therapy for H. pylori (see above)
10. surgical tx
1. indications: I HOP

I = Intractability

H = Hemorrhage (massive or relentless)

Note: exsanguination is the most common cause of death from a duodenal ulcer

O = Obstruction (gastric outlet obstruction)

P = Perforation

2. goal: decrease gastric acid secretion and fix IHOP
3. options for Intractability

1. highly selective vagotomy (preserves pyloric branches, operation of choice b/c only 0.5% mortality, but increased recurrence, 10-25%)
2. vagotomy and pyloroplasty (0.5-1% mortality, 10-12% recurrence)
3. vagotomy and antrectomy Billroth I or II (1-2% mortality, 1-2% recurrence) (see Surg Recall p.235 for Billroth I, II explanations)

4. for Hemorrhage, open duodenum through pylorus, oversew bleeding vessel, perform vagotomy and pyloroplasty
5. options for Obstruction

1. truncal vagotomy, antrectomy, and gastroduodenostomy (Billroth I) or gastrojejunostomy (Billroth II)
2. truncal vagotomy and drainage procedure (gastrojejunostomy)

6. options for Perforation

1. Graham patch for poor candidates, shock, or prolonged perforation (Graham patch is a piece of omentum incorporated into the suture closure of a perforation)
2. truncal vagotomy and pyloroplasty, incorporating ulcer
3. Graham patch and highly selective vagotomy
4. truncal vagotomy and antrectomy

11. complications
1. early

1. hemorrhage (may need to reoperate)
2. gastric retention from vagotomy
3. duodenal stump leakage with Bilroth II (must usually reoperate)

2. late

1. dumping: ealy dumping: uncontrolled emptying of hypertonic fluid from intravascular space to intraluminal space, leading to intravascular volume depletion and diarrhea; late dumping: rapid absorption of glucose due to improper gastric emptying, leads to large insulin increase – over-correction of blood glucose leads to transient hypoglycemia approx. 3 hrs after meal

2. diarrhea (tx with Lomotil, Kaopectate, Imodium)

3. reflux gastritis (convert Billroth I/II to gastrojejunostomy)

4. gallstones
5. weight loss
6. iron/vit B-12 deficiency due to decreased intrinsic factor secretion (tx with supplementation)

-----------------------------------------------------------------------------------------------------------

18- In triage, green color indicates

a. Ambulatory patients
b. Dead/Moribund patients
c. High priority treatment or transfer
d. Medium priority or transfer

-----------------------------------------------------------------

The correct answer is A

TRIAGE TAG Color Code Priorities


1. RED--- Immediate
1. Serious but salvageable, with life-threatening injuries. Severe burns, bleeding, impaired breathing and internal injuries.
2. Red Tagged patients are transported first from Casualty Collection Area.
3. Examples of Injuries:
1. Witnessed Cardiac Arrest
2. Uncorrected Respiratory Problems (not Minor Distress)
3. Severe Bleeding and Shock
4. Open Chest and Abdominal Injuries
5. Major Fractures and Burns (Full Thickness/ Airway)
6. Unconscious Patients
7. Severe Medical Problems (Heart Attack, Poisoning)
8. Injured Co-workers and Severe Emotional Disorders


2. YELLOW--- Moderate
1. Moderate to serious injuries. Victims with potentially serious injuries such as long bone fractures and moderate bleeding are assigned here.
2. Yellow tagged patients are transported immediately after Red Tag and may be transported with a green tag patient.
3. Examples of Injuries:
1. Severe Burns not affecting the airway
2. Spinal Injuries
3. Moderate Blood Loss
4. Head Injuries


3. GREEN--- Minor
1. “Walking Wounded” and minor injuries. Patient is not seriously injured.
2. Delayed treatment and transport
3. Example of Injuries:
1. Minor Injuries- Cuts and Abrasions
2. Minor Fractures
3. Mortal Injuries where death appears imminent


4. BLACK--- Deceased
1. Victims who are found to be obviously deceased with no vital signs or obviously fatal injuries.
2. Transport arrangements are made with Coroner.
3. Patients should not be moved by EMS Personnel, as they are part of the crime scene.


5. WHITE--- Non-Injured
1. For patient accountability
2. Person was involved in the incident but is not injured.
3. Transportation will occur when available and by mass transit capabilities.

--------------------------------------------------

19- Chlamydia trachomatis infection causes the following EXCEPT

a. Pneumonitis
b. Rhinitis
c. Conjunctivitis
d. Urethritis

----------------------------------

The correct answer is B

* Chlamydiae are very small bacteria which have to live intracellularly.
* They were originally considered to be viruses, but it is now known that they contain both DNA and RNA and are structurally related to Gram-negative bacteria.
* Several species are known in the genus Chlamydia: C. psittaci, the pathogen of psittacosis; C. pneumoniae (the old TWAR), pathogen of atypical pneumonia; and C. trachomatis, which has many serotypes.
* Serotypes A, B, Ba, and C cause trachoma. Serotypes D to K cause inclusion conjunctivitis in the newborn (“paratrachoma”), Reiter’s syndrome, non-gonococcal urethritis, epididymitis, cervicitis and P.I.D. (pelvic inflammatory disease). Neonatal conjunctivitis and pneumonia can be caused in the newborn by these bacteria. Serotypes L1 and L2 cause the sexually-transmitted disease lymphogranuloma venereum. L3 causes pneumonia in mice.
* C. trachomatis is considered to be responsible for 20% of the pharyngitis symptoms in adults.

---------------------------------------------------

20- Which of the following types of nerve fibres carry pain?

a. A alpha
b. A beta
c. A gamma
d. A delta

-------------------------------------------------

The correct answer is D


A-alpha fibers - the largest myelinated fibers, range in diameter from 7-16 microns,conduction velocity from 70-120m/s, encoded for the transmission of muscle spindle and tendon organ afferents

A-beta fibers - responsible for sensibility of touch, diameter from 6-8 microns, conduction velocity 30-70 m/s.

A-delta fibers - transmit stimuli encoded for temperature and fast or first pain, smallest myelinated, diameter 2.5-4 microns, conduction velocity 12-30 m/s

Unmyelinated C fibers - transmit stimuli encoded for slow or second pain, temperature, efferent sympathetic fibers, conduction velocity 0.5-2 m/s.

----------------------------------------------------
21- Vossius ring is seen in

a. Cornea
b. Lens
c. Vitreous
d. Retina

---------------------------------------------------

The correct answer is B


Traumatic Cataract

* Unilateral
* Rosette-shaped
* Can be lamellar
*

Vossius' ring is a traumatic deposit of iris pigment epithelium on the anterior lens capsule and usually fades with time.

--------------------------------------------------------

22 - Anti histone antibodies are characteristic of

a. drug induced lupus
b. cardiac lupus
c. lupus nephritis
d. MCTD

--------------------------------------------------------------

The correct answer is A

DRUG INDUCED SLE

* Most common drug is procainamide, which induces ANA in 50-75% of individuals within a few month

o 20% of these patients develop a clinical drug induced lupus, which may consist of polyarthralgias, systemic symptoms, anemia, leukopenia, thrombocytopenia, rheumatoid factors, false positive VDRL and positive coombs test.

* Hydralazine is another common drug that induces ANA in 25-30% of patients and lupus like symptoms in 10% of patients.

* The major serological differences between drug induced lupus and non-drug induced lupus is that antihistone bodies are more common in drug induced lupus while it is very rare to find anti double stranded antibodies on Smith antigen in drug induced lupus.

* Identical to SLE except when take the drug away, patient gets better (duh)

* Linkage with HLA-DR4

--------------------------------------------------------------------------

23-Sump syndrome occurs most commonly after

a. Cholecystojejinostomy
b. Choledochoduodenostomy
c. Mirizzis syndrome
d. Choledochojejunostomy

----------------------------------------------------------

The correct answer is B

Sump syndrome: -

A complication of side-to-side choledocho-duodenostomy, in which the lower end of the CBD

At times acts as a diverticulum, resulting in stasis, trapping of food particles, and infection.

Often the same symptoms that are present before the operation recur or postoperative symptoms such as: Colicky pain, bloating, nausea, and vomiting occur. There is pain on palpation of the RUQ and sometimes jaundice.

---------------------------------------------------------

24-In Bennet's Fracture, the following bone is injured

a. 1st metacarpal
b. 2nd metcarpal
c. 3rd metacarpal
d. 4th metacarpal

------------------------------------------------------------

The correct answer is A

A Bennett's fracture is an intra-articular fracture of the proximal end (base) of the thumb metacarpal. The resulting bone fragment is held by the intermetacarpal ligament. The base of the metacarpal is displaced laterally by the pull of the abductor pollicis longus

---------------------------------------------------------------------

25-One of the following statement isfalse regarding complications of Protein Energy Malnutrition

a. Hyperglycemia
b. Hypothermia
c. Septic shock
d. Electrolyte imbalance

------------------------------------------------------

The correct answer is A

Malnutrition is any physical condition resulting either from an inappropriate or inadequate diet, such as a diet that either provides too much or too little of necessary nutrients, or from a physical inability to absorb or metabolise nutrients.

Complications

(1) Anemia

(2) Vitamins deficiency

(3) Infections

(4) Spontaneous hypoglycemia

(5) Disorder of water and electrolyte

-------------------------------------------------------------------

26-The activities of all the following enzymes are increased in starvation EXCEPT

a. Pyruvate kinase
b. Pyruvate carboxylase
c. Phosphoenolpyruvate carboxykinase
d. Glucose-6-phosphatase

-----------------------------------------------------------

The correct answer is A


The activities of glucose-6-phosphatase (G6Pase), fructose-1,6-bis-phosphatase(FDPase), pyruvate carboxylase (PC) and phosphoenolpyruvate carboxykinase (PEPCK), the key gluconeogenic enzymes, increase markedly, indicating compensatory gluconeogenesis.

-------------------------------------------------------------------

27-common congenital anomaly of midgut is

a. Hirschsprung's disease
b. Oomphalocele
c. Duodenal atresia
d. Meckel's Diverticulum

--------------------------------------------------------

The correct answer is D

Meckel's diverticulum is the most common congenital anomaly of the gastrointestinal tract, affecting about 2% of the population. Meckel's diverticulum affects the distal ileum and represents the remnants of the proximal end of the embryologic yolk stalk (i.e., the omphalomesenteric or vitelline duct) which normally obliterates completely by the 8th week of gestation.

---------------------------------------------------------

28-There is no criminal responsibility below the age of

a. 5 yrs
b. 7 yrs
c. 10 yrs
d. 12 yrs

-----------------------------------------------------

The correct answer is B


Section 83 of the Indian Penal Code, according to which, nothing is an offence which is done by a child above seven years of age and under 12 years, who has not attained sufficient maturity of understanding to judge the nature and consequences of his conduct on that occasion.

It may be noted that children below the age of seven years are deemed to be incapable of criminal offence as per section 82 of the Indian Penal Code.

----------------------------------------------------

29-Hepatitis C Virus resembles to whichof the following virus groups?

a. Picrona
b. Herpes
c. Hepadna
d. Flavi

--------------------------------------------

The correct answer is D

The Hepatitis C virus (HCV) is a small (50 nm in size), enveloped, single-stranded, positive sense RNA virus in the family Flaviviridae.

------------------------------------------------------

30-Arthus phenomenon is an example of which hypersensitivity?

a. Type I
b. Type II
c. Type III
d. Type IV

---------------------------------------------------------

The correct answer is C

Arthus reaction demonstrated by Maurice Arthus in 1903.


1. Artificially induced in laboratory.

2. Induced localized inflammatory skin reaction in previously sensitized rabbit (to horse serum) by intradermal injection of cognate Antigen.

Inflammation grossly visible after several hours

a. Longer time than Type I, but shorter than Type IV. Classified as immediate because responsible antibodies can be passively transferred via serum.

b. Ag-Ab complexes form and adhere to vascular endothelium; complement system activated and some of its chemotactic intermediates attract neutrophils to site. Anaphylatoxins degranulate mast cells with resultant release of histamine that causes constriction of arterioles and retards blood supply to area. Platelets stimulated by the immune complex initiate the coagulation cascade, resulting in fibrin deposits. Eventually vessels become clogged with thrombin and accumulated cells, causing an exudate into the surrounding tissue (edema). Deprivation of blood supply to area results in ischemic necrosis.

c. Only precipitating (multivalent) antibodies can elicit the Arthus reaction (mainly IgG). Relatively large amounts of antigen required.

d. Reverse passive Arthus possible (Ab from sensitized animal injected intradermally, Ag IV or at site).

----------------------------------------------------------------------

31- Cholecalciferol produces all of the following effects EXCEPT

a. Synthesis of new calcium binding proteins
b. Induction of osteocalcin for bone mineralisation
c. Regulation of paratyroid secretion
d. Increase calcium and phosphate excretion

---------------------------------------------

The correct answer is D


Cholecalciferol is Vitamin D

Vitamin D is very important to the health of bone. 1,25- dihydroxy-D3 stimulates the production of a protein in the digestive system called calbindin. Calbindin allows the body to absorb calcium from the digestive system into the blood stream.

---------------------------------------------------------

32- Cutaneous vasoconstriction is mediated by

a. Sympathetic adrenergic nerves
b. Sympathetic cholinergic nerves
c. Parasympathetic cholenergics
d. Somatic nerves

-------------------------------------------------

The correct answer is A

VASCULAR (ARTERIOLAR) CONTROL

Mainly via the sympathetic nervous system: tonic discharge of sympathetic nerves enhances the basal tone of the arterioles (via neurally released noradrenaline, NAd). There is, with a few notable exceptions, little parasympathetic innervation of arterioles.

Change in this the sympathetic discharge rate either vasoconstricts or vasodilates arterioles:

↑ sympath. activity → arteriolar vasoconstriction

↓ sympath. activity → arteriolar vasodilatation

This forms part of reflex control of the CVS to regulate MAP.

Because adrenergic influences are mediated by α-adrenoceptors in the VSM (controls TPR), and β-adrenoceptors in the heart (controls CO), selective α or β blockers can be used to block one or the other.

--------------------------------------------------------------

33- Serum creatine kinase-3 (CK-3) is elevated in

a. Muscular dystrophy
b. Myocardial infarction
c. Alcoholic cirrhosis
d. Brain tumors

----------------------------------------------------------

The correct answer is A

Creatinine kinase (CK) is an enzyme, found primarily in muscle and brain tissue, which exists as three dimeric isoenzymes — CKMM (CK-3), CK-MB (CK-2), and CK-BB (CK-1) — built from subunits designated M and B.

The CK-MB isoenzyme, which has a molecular mass of approximately 87,000 daltons, accounts for 5 to 50% of total CK activity in myocardium.

In skeletal muscle, by contrast, it normally accounts for just 1% or less, CK-MM being the dominant form, though the percentage can be as high as 10% in conditions reflecting skeletal muscle injury and regeneration (e.g. severe exercise, muscular dystrophy, polymyositis).

--------------------------------------------------------------------

34- What is the standard dose of PPD for Mantoux test?

a. 5 TU
b. 10TU
c. 15 TU
d. 20 TU

------------------------------------------------------------------

The correct answer is A


A standard dose of 5 Tuberculin units (0.1 mL)is injected intradermally (into the skin) and read 48 to 72 hours later. A person who has been exposed to the bacteria is expected to mount an immune response in the skin containing the bacterial proteins.

The reaction is read by measuring the diameter of induration (palpable raised hardened area) across the forearm (perpendicular to the long axis) in millimeters. No induration should be recorded as "0 mm". Erythema (redness) should not be measured.

If a person has had a history of a positive tuberculin skin test, another skin test is not needed.

-----------------------------------------------------------

35- Histrionic personality disorder is recognized by

a. Withdrawn behaviour
b. Extrovert with excessive emotionality and attention seeking
c. Extrovert with emotional warmthness
d. Obsessie features

-------------------------------------------------

The correct answer is B


Histrionic Personality Disorder

Pervasive pattern of excessive emotionality, attention-seeking as demonstrated by at least 5 of:

1. uncomfortable if not the center of attention
2. inappropriately seductive
3. rapid, shallow expression of emotion
4. use of appearance to draw attention
5. impressionistic speech style
6. expressions of emotions exaggerated
7. unable to delay gratification
8. considers relationships more intimate than they are

Prevalence: 2-3% general population. Male: Female, slightly more female (as in general population); gay persona, “macho” stance. 10-15% psych outpatients.

Etiology: usually an over sexualized, over stimulated, “special” child; NOT always frankly abused.

Differential: hypomania, brain tumor/infarct (Klüver-Bucy), culture, adaptation/manipulation.
Comorbidity: late phase depression, Somatization, substance problems, other PD.

---------------------------------------------------------------------

36- The competence of the deep venous system is diagnosed clinically by performing which of the following tests

a. Pratt's test
b. Scwartz's test
c. Perthe's test
d. Brodie-Trendelenberg's test

----------------------------------------------------------

The correct answer is C

Perthe's test - Checks competence of the venous valves of lower extremities.


# Release a bit of tourniquet, but pt raises up and down on toes after releasing.
# If perforating calf veins have competent valves, calf pump will function, making varicosities less tense.

----------------------------------------------------

37- The drug of choice for persistent unstable ventricular arrythmia is

a. procainamide
b. Calcium gluconate
c. Amiadarone
d. Digoxin

----------------------------------------------

The correct answer is C

Like all antiarrhythmic drugs, amiodarone can aggravate arrhythmia; this is reported in 3% to 5% of patients.However, because of the long time course of drug action, it may be difficult to distinguish between the natural history of the underlying cardiac disease and the arrhythmia and a drug-related aggravation of arrhythmia. Although amiodarone markedly prolongs the QT interval, torsade de pointes is an infrequent complication and is most often reported in association with hypokalemia or with concomitant therapy using a class IA drug, especially quinidine.

-----------------------------------------------------------

38 - In adrenogenital syndrome the diagnostic criteria are following EXCEPT

a. Karyotyping 46 XX
b. Serum 17-hydroxy progesterone is normal
c. Urinary pregnenetriol is increased
d. Normalinternal genital organs

-----------------------------------------------

The correct answer is B

Congenital Adrenal Hyperplasia (CAH)

* genetic disorder which involves deficiency in one of enzymes required in steroid hormone synthesis
* in enzyme defects associated with cortisol deficiency, hyperplasia of adrenal gland may occur due to excessive stimulation by ACTH
* when enzyme deficiency leads to virilisation of affected females (masculinisation of ext genitalia) disorder sometimes referred to as adreno-genital syndrome

o wolffian duct development consistently absent in virilised females, who, therefore, have normal internal sex organs
o why development is not stimulated by excess androgen is not understood, but high local concentrations created by testes may be required

The most common block is the 21 Hydroxylase block, which accounts for more than 90% of all CAH cases. The steroid that accumulates is 17 Hydroxy progesterone. This is an easy diagnostic test to do in the newborn period. The blood spot, which is taken at 5 days, can be tested for 17 Hydroxy-progesterone.

-------------------------------------------------------------------

39 - Toxic shock syndrome is due to

a. Septic abortion
b. Forgotten tampons
c. IUCD
d. Pelvic examination

-------------------------------------------------------------

The correct answer is B

Toxic shock syndrome (TSS) is an illness marked by acute onset of fever, hypotension, rash, desquamation and failure of multiple organ systems. It is caused by a bacterial infection, predominantly Staphylococcus aureus, but also commonly caused by group A Streptococcus. Cases of TSS caused by other genera of bacteria (e.g. Clostridium sp.) have been reported as well.

Toxic shock syndrome is best known for its association with tampon use (menstrual toxic shock syndrome); however, there are many other causes of infection.

Transmission of staphylococcal TSS requires colonization with the bacteria, usually on a fomite placed inside the body or directly as in the case of burn patients whose wounds are susceptible to infection.

Toxic shock syndrome can be treated with antibiotics; antibiotic prophylaxis has been shown to be effective in high-risk burn patients

-----------------------------------------------------------------

40 - The most accurate method of diagnosis of gastroesophageal reflux disease is

a. Barium swallow
b. Endoscopy
c. 24 hour pH recording
d. Esophageal manometry

-------------------------------------------------------

The correct answer is C


24-hr pH Monitoring

* Registers the amount and frequency of acid in the esophagus and allows correlation with symptoms such as heartburn and pain. A probe is placed into the esophagus which records the acid level in both the esophagus and stomach for a full 24 hours.
* This is the most accurate method of detecting reflux and GERD

------------------------------------------------------------

41 - The most common complication of transurethral resection of the prostate is

a. Erectile dysfunction
b. Retrograde ejaculation
c. Urinary incontinence
d. Urethral stricture disease

---------------------------------------------------------------

The correct answer is D

Urethral stricture is the most common complication of transurethral resection of prostate, occurring in up to 29 per cent of cases.

----------------------------------------------------------------------

42 - The commonest anterior mediastinal tumor is

a. Aneurysm of ascending aorta
b. Neurogenic tumor
c. Thymoma
d. Bronchogenic cyst

-------------------------------------------------------------

The correct answer is C


Thymoma is not uncommon in adults, and accounts for 12% of all primary mediastinal masses. The average age at onset is the fifth decade of life and the disease is rare in children. It is the commonest tumor of the anterior mediastinum and is locally invasive in 30-50% of cases.

Thymoma involves mainly the anterior mediastinum and may rarely arise from thymic rests which are not infrequently encountered in the pericardium.

-------------------------------------------------------

43 - Koplik spots is a feature of

a. Rubella
b. Measles
c. Mumps
d. Tetanus

--------------------------------------------------

The correct answer is B

--------------------------------------------------

44 - Gunstock deformity occurs in

a. Supracondylar fracture of femur
b. Intercondylar fracture of femur
c. supracondylar fracture of humerus
d. Distal radial fractures

---------------------------------------------------

The correct answer is C

A fracture of the lower humerus besides leaving a valgus deformity, may cause the opposite or a cubitus varus. This is known colloquilly as a gunstock deformity. It may require a osteotomy of the lower humerus to correct this in the future.

------------------------------------------------------

45 - Drug of choice for methicillin resistant staphylococci aureus is

a. Ceftriaxone
b. Ceftazidime
c. Ampicillin
d. Vancomycin

----------------------------------------------------------

The correct answer is D

MRSA is a germ that can cause Staph infection. It is resistant to most antibiotics including methicillin, the longtime drug of choice for treating many common Staph infections.

The antibiotic vancomycin is given intravenously for MRSA infections.

-------------------------------------------------------

46 - In Erb-Duchene paralysis, the injury is limited to

a. 2nd and 3rd cranial nerves
b. 3rd and 4th cranial nerves
c. 4th and 5th cranial nerves
d. 5th and 6th cranial nerves

------------------------------------------------------------

The correct answer is d

Erb-Duchene paralysis: Waiters Tip

C5,6

* Nerve to Subcavius (ST)
* Suprascapular (ST)
* Lower Subscapular (PC)
* Upper Subscapular (PC)
* Axillary

------------------------------------------------------------------

47 - Cobblestone appearance is seen in

a. Ulcerative colitis
b. Chron's disease
c. Appendicitis
d. Carcinoma rectum

-----------------------------------------------------

The correct answer is B

Endoscopic features of Crohn’s disease include: aphthous ulcers, irregular punched-out or longitudinal “serpiginous” ulcers, pseudopolyps, inflammatory polyps, “cobblestone” appearance of the mucosa, strictures, and fistula. Pathologic features of Crohn’s disease (based on biopsy specimens) include many of the same findings as with ulcerative colitis, but the inflammation is transmural, goblet cells and mucin are usually preserved, and granulomas may be present. A granuloma is pathognomonic for Crohn’s disease, but is only present in 50% of cases and its absence does not exclude the diagnosis of Crohn’s disease.

The endoscopic appearance in ulcerative colitis is likened to “wet sandpaper.” Deep, large ulcers (macro-ulceration) of the mucosa may be present in severe ulcerative colitis, but in the majority of people with ulcerative colitis, endoscopically apparent ulcers are usually not seen (in contrast to Crohn’s disease).

----------------------------------------------------------------------

48 - Classical 'raindrop' lesions are seen in

a. Burkitt's lymphoma
b. Hodgkin's lymphoma
c. Multiple myeloma
d. Haemophilia

------------------------------------------


The correct answer is C

Multiple Myeloma

Key clinical features

1. 75% of patients are between 50-70 yoa and 2:1 male preponderance
2. LB Pain is the cardinal initial symptom relieved with bedrest and aggravated with weight. With sciatica
3. Bacterial infections occur in 10% of cases most respiratory in nature.
4. Pathological fracture is common complication

Key radiologic findings

1. Bone Scans are cold
2. Gross osteoporosis may be the only early sign
3. Punched lesions are the radiologic hallmark of myeloma
4. Vertebra plana or wrinkled vertebra is characteristic.
5. Raindrop skull (lytic myeloma defects) and pedicle sign of myeloma (preservation of pedicles) occur.
6. “Moth Eaten,” Permeative lesions
7. Foggy cortical – Medullary border

Key laboratory features

1. 40% show Bence Jones’ proteinuria.
2. Immnoglobulin Electrophoresis – to Elevation of IgG
3. May have infections due to decreased WBCs
4. Normocytic, normochromic anemia

Well-defined osteolytic areas are visible in the bone of the calvaria (classical 'raindrop' lesions) (arrows).



------------------------------------------------------------------

49 - Dennie-Morgan fold is seen in

a. Dermatomyositis
b. Systemic lupus erythematosus
c. Psoriasis vulgaris
d. Atopic dermatitis
-------------------------------------------------

The correct answer is D


Atopic Dermatitis

* 10% of kids in industrialized nations - incidence rapidly increasing
* Onset almost always in infancy, and 60-70% will continue to have some manifestations throughout life
* A large % will develop other atopic diseases and many have family history of atopy

Clinical Findings

* Severe pruritis, interferes with sleep. Dbn varies with age
* 3 mo: scalp, face, extensor surfaces, i.e. where they can rub (diaper spared)
* 2 yrs: flexural involvement, neck, antecubital and popliteal fossa, periorbital area, and hands/feet - leading to more chronic lichenification
* Adults: similar, including hands, nipple, periorbital
* Associated features include dry skin
* ichthyosis vulgaris - auto-dom
* abn keratinazation
* keratosis pilaris - rough follicular pustules on posterolateral aspects of arms, thighs
* pityriasis alba - poorly circumscribed hypopigmented macules
* Dennie-Morgan fold - in eyelid, from rubbing
* Increased risk for Staph aureus, viral, and fungal infections (impaired cell-med immunity)

Pathogenesis

* Uncertain, likely to be polygenic
* Linked mutations involve genes that regulate the immune response
* A TH2 dominated disease with increased amounts of IL4, 5, 10 in the skin and blood. Accopanied by increased amounts of IgE and eosinophilia.

Treatment

1. Moisturize the skin
2. Control itching -antihistamines
3. Decrease inflammation -topical glucocorticoids

Pts w/ severe disease require systemic immunosuppression

--------------------------------------------------------------

50 - Rigler's sign is seen in

a. ulcerative colitis
b. Chron's disease
c. Megacolon
d. pneumoperitoneum

----------------------------------------------

The correct answer is D

Rigler's sign, also known as the double wall sign, is seen on an x-ray of the abdomen when air is present on both sides of the intestine; a Rigler's sign is present when air is present on the inside (lumenal side) and the outside (peritoneal side).

-------------------------------------------------

51- The first modality of sensation to be lost in leprosy is

a. Touch
b. Pain
c. Temparature
d. Joint sense

-------------------------------------------------

The correct answer is C

Temperature is the first sensation that is lost. Patients cannot sense extremes of hot or cold. The next sensation lost is light touch, then pain, and finally deep pressure. These losses are especially apparent in the hands and feet; therefore, the chief complaint may be a burn or ulcer in an anesthetic extremity.

---------------------------------------------------------

52 - Carbon dioxide transported in blood mostly as

a. Carboxyhemoglobin
b. In combination with plasma proteins
c. Bicarbonate
d. Carbonic acid dissolved in plasma

--------------------------------------------------------------

The correct answer is C

Carbon Dioxide Transport

* Carbon dioxide is transported in the blood in three forms
* Dissolved in plasma – 7 to 10%
* Chemically bound to hemoglobin – 20% is carried in RBCs as carbaminohemoglobin
* Bicarbonate ion in plasma – 70% is transported as bicarbonate (HCO3–)

------------------------------------------------------------------

53 - All the following are signs of cerebellar disease EXCEPT

a. Resting tremors
b. Past pointing
c. Nystagmus
d. Ataxic gait

-------------------------------------------------------------------

The correct answer is A

Signs Of Cerebellar Disease

* gait ataxia
* disturbed stance
* limb dysmetria
* limb dysynergia
* kinetic (intention) tremors
* dysarthria
* past-pointing
* excessive rebound
* impaired check
* dysdiadochokinesis
* head tilt or titubation
* hypotonia
* nystagmus
* ocular dysmetria or ocular dysynergia
* gaze palsies

----------------------------------------------------------------------

54 - All the following factors affect the APGAR score EXCEPT:

a. Prematurity
b. Maternal sedation
c. Neurological condition of the baby
d. Mode of delivery

-----------------------------------------------------------------------------------

The correct answer is D

APGAR SCORE: a composite measure of the physical health of an infant, assessed at one and five minutes after birth, and used to predict the infant’s chances of survival. Five easily identifiable characteristics of the baby are assessed on a scale of 0 to 2, with 2 being optimum: heart rate, respiratory effort, muscle tone, reflex irritability, and color. The Apgar score is the sum of these scores, with a score of 7 or higher indicating that the infant is in good physical condition, and a score of 10 being perfect.

Factors affecting apgar score:

* Prematurity
* Drugs-sedatives,narcotics,mgso4
* A/c cerebral trauma
* Precipitate labor
* Cong. Myopathy
* Cong. Neuropathy
* Spinal cord trauma
* CNS anomaly
* Lungs-diaphragmatic hernia
* Airway-choanal atresia
* Cong. Pneumonia (GBS)


* Maternal acidosis
* High fetal catecholamine levels
* Some full term infants

---------------------------------------------------------------------------------

55 - The commonest cause of acute pancreatitis is

a. Biliary calculi
b. Alcohol abuse
c. Infective
d. Idiopathic

-------------------------------------------------------------------------------------

The correct answer is A


The commonest cause of severe acute pancreatitis is blockage of the pancreatic duct by gallstones. This can sometimes occur even if the gallbladder has been previously removed. When triggered by excessive alcohol consumption, acute pancreatitis usually resolves itself with rest and abstinence from drinking.

----------------------------------------------------------------

56 - Failure to initiate and maintain spontaneous respiration following birth is clinically known as

a. Birth asphyxia
b. RDS- Respiratory Distress Syndrome
c. Respiratory failure
d. Pulmonary oedema

-------------------------------------------------------------------

The correct answer is A

WHO has defined perinatal asphyxia as a “failure to initiate and sustain breathing at birth”.

Moderate Birth asphyxia was defined as “Slow gasping breathing or an Apgar score of 4-6 at 1 minute of age.

Severe Birth asphyxia was defined as no breathing or an Apgar Score of 0-3 at 1 minute of age.

-------------------------------------------------------------

57- The Meckel's diverticulum is situated within about __________ cms from the ileocecal valve

a. 25 cms
b. 60
c. 75
d. 100

--------------------------------------------------------------------

The correct answer is B

Meckel's diverticulum is a true diverticulum containing all layers of the intestinal wall, usually arising from the antimesenteric border of the ileum 45–90 cm. proximal to the ileocecal valve. It is a vestige of the omphalomesenteric or vitelline duct, which usually undergoes complete obliteration during the seventh week of gestation. Autopsy studies have estimated the incidence of Meckel's diverticulum to be 1% to 2% with men being more commonly affected than women by a ratio of 2:1. Gastric mucosa is present in 50% of all Meckel's diverticula, but in over 75% of symptomatic individuals.

----------------------------------------------------------

58 - Commonest site of otosclerosis is

a. Round window
b. Oval window
c. Utricle
d. Ossicles

------------------------------------------------------------

The correct answer is B

New spongy bone is laid down around the footplate of the stapes which impedes its ability to move freely in the oval window. This causes a conductive deafness which gets worse as the disease process develops.

Examination is normally normal although in very active disease it may be possible to see a pink hue to the drum, from very vascular spongy bone deposits on the medial wall of the middle ear. (Schwarze's sign).

----------------------------------------------

59 - ASO (Antistreptolysin O) test is used for the diagnosis of

a. Rheumatoid Arthritis
b. Typhoid fever
c. Rheumatic Fever
d. Rickettsial fever

------------------------------------------------------------

The correct answer is C

Antistreptolysin O antibodies will be raised after infection with streptococci. Levels greater than 200 units per millilitre are considered significant (although cut off levels will vary between laboratory undertaking the test).

Measurement can be useful diagnostically if recent infection with streptococci is known to be important in aetiology. Diseases include:

* rheumatic fever
* post streptococcal glomerulonephritis
* scarlet fever
* erysipelas

----------------------------------------------------------------

60 - Antiglomerular Basement Membrane Antibodies are seen in

a. Goodpasture syndrome
b. Henoch-Schonlein pupura
c. Heymann nephritis
d. Systemic Lupus erythematosus

--------------------------------------------------

The correct answer is A

Goodpasture syndrome (antiglomerular basement membrane disease)

Ña. The cause is antibodies (antiglomerular basement membrane antibodies) directed against antigens in glomerular and pulmonary alveolar basement membranes.

Ñb. Fluorescent antibody studies for IgG demonstrate linear immunofluorescence.

c. Clinical manifestations include:

Ñ(1) Nephritic syndrome

Ñ(2) Pneumonitis with hemoptysis (hemorrhagic pneumonitis)

(3) Peak incidence in men in their mid-20s

(4) RPGN crescentic morphology with linear immunofluorescence

--------------------------------------------------------------

61 - Onion skin pattern of cortical bone formed by tumor is characteristic of

a. Giant cell tumor
b. Osteosarcoma
c. Chondrosarcoma
d. Ewing sarcoma

-----------------------------------------------------------

The correct answer is D

Ewing’s sarcoma

* Is a primitive primary malignant tumor of bone it is composed of tumor cells derived from the connective tissue framework of bone marrow


1. 7% of all primary bone tumors. 4th m/c primary malignant bone tumor behind (multiple myeloma, osteosarcoma, and chondrosarcoma).
2. 10-25 years of age


2. Key clinical features
1. Mimics: Infection; systemic signs of slight fever, secondary anemia leukocytosis, and increase ESR
2. Affects long tubular bones of the Lower extremity.


3. Key radiologic features
1. Saucerization: Cortical saucerization is a characteristic sign. This irregular defect effaces the outside of the bone occasionally, exhibiting a marginal scalloping effect.
2. Classic presentation is a diaphyseal permeative lesion with a delicate onion skin or peel periosteal response.
3. The most common primary malignant bone tumor to metastasize to bone

-------------------------------------------------------------------------------

62 - Fracture dislocation of 2nd and 3rd cervical vertebrae is common in

a. Partial hanging
b. lynching
c. Judicial hanging
d. throttling

-----------------------------------------------------------------------------

The correct answer is C


In judicial hanging the posterior arch of the axis is snapped clean off and remains fixed to the third vertebra, while the atlas, the Odontoid process, and the anterior arch of the axis remain fixed to skull.

Schneider and colleagues first introduced the term hangman fracture into the modern medical lexicon in 1965 when they described eight patients who sustained bilateral fractures through the neural arch of the axis, with or without dislocation of the body of the axis from that of the third cervical vertebra. According to these authors, "This type of fracture-dislocation occurs in modern judicial hanging and in some instances of traffic accidents".

Most likely, the forces involved in judicial hanging are much greater than those sustained by many motor vehicle accident victims, and the spinal disruption between C2 and C3 caused by judicial hanging is 3 -considerably greater.

-----------------------------------------------------------------

63 - All are features of Plummer-vinson syndrome EXCEPT

a. Oesophageal web
b. Iron deficiency
c. Achalasia Cardia
d. Dysphagia

--------------------------------------------------------------------------

The correct answer is C

Plummer-Vinson syndrome: hypopharyngeal web + iron-deficiency anemia in middle-aged women. This syndrome occurs most commonly in individuals of Scandinavian descent and is characterized by esophageal webs, iron deficiency anemia, dysphagia, achlorhydria, atrophic gastritis, hiatal hernia, and increased risk of cancer. Dysphagia improves in the Plummer-Vinson syndrome with iron therapy, though the webs remain.

-------------------------------------------------------

64 - The earliest symptom in primary biliary cirrhosis is

a. Jaundice
b. Pruritis
c. Melanosis
d. Vomiting

---------------------------------------------------------------------

The correct answer is B

Primary biliary cirrhosis (PBC) is a disease characterized by inflammatory destruction of the small bile ducts within the liver. PBC eventually leads to cirrhosis of the liver.

Most patients with PBC present with pruritus (itching). After pruritus, jaundice (yellow skin caused by bilirubin retention) is the most common presenting symptom.

-------------------------------------------------------

65 - The drug used in the treatment of complications of multiple sclerosis is

a. Rifampicin
b. Isoniazid
c. Ethambutol
d. Streptomycin

------------------------------------------------------------

The correct answer is B

A drug normally used to fight tuberculosis, isoniazid (INH) with pyridoxine, is also helpful in controlling severe gross tremors which are posture related. Unfortunately, large doses of isoniazid are needed to be effective, and this can result in liver toxicity. Given the danger, this medication should only be used in extreme cases, and liver function must be evaluated regularly.

Four patients with disabling action tremor in the setting of MS were treated with isoniazid (800 to 1200 mg per day). All patients showed significant improvement of the tremor, allowing more functional use of their extremities. To the best of our knowledge, this is the only medical treatment for this type of tremor.

Ref: Treatment of action tremor in multiple sclerosis with isoniazid;A. F. Sabra, M. Hallett, L. Sudarsky and W. Mullally ; American Academy of Neurology ;NEUROLOGY 1982;32:912

-----------------------------------------------------------------

66- Amphotericin may be administered mixed in any of the following EXCEPT

a. Glucose infusion
b. Mannitol infusion
c. Saline infusion
d. Lipid- complex (ABLC)

-------------------------------------------------------------

The correct answer is D

The combination of amphotericin B and intralipid should not be given to patients

-----------------------------------------------------------------

67 - Which of the following response is found in atopic individuals?

a. IgM reponse
b. IgE response
c. Ig A response
d. Hypogammaglobulinemia

---------------------------------------------------------------

The correct answer is B


The immune system of mammals makes receptor proteins (antibodies) to substances that are foreign (i.e. not part of the body), each antibody being specific to a given substance. Antibodies are of several types, IgG for instance being involved in protection against viral diseases after vaccination whereas IgE, involved in atopic individuals, is particularly concerned with protection against parasites.

-----------------------------------------------------------

68- Tetanus toxoid given during antenatal period will protect

a. Only the child
b. Only the mother
c. Both mother and child
d. Partially

-------------------------------------------------------

The correct answer is C

Tetanus spores infect women and children through unsafe or unclean deliveries, accounting for an estimated 5% of maternal deaths and 14% of neonatal deaths.Tetanus toxoid vaccines can prevent infections and save the lives of mothers and infants alike. Pregnant women should receive at least two doses of tetanus toxoid, which provide one to three years of protection.

------------------------------------------------------------

69 - Fleischer ring is seen in

a. Tay Sachs disease
b. Keratoconus
c. Siderosis Bulbi
d. Contusion injury

------------------------------------------------------------

The correct answer is B

Fleischer rings are pigmented rings in the peripheral cornea, resulting from iron deposition in basal epithelial cells. They are usually yellowish to dark-brown, and may be complete or broken.

Fleischer rings are indicative of keratoconus, a degenerative corneal condition that causes the cornea to thin and change to a conic shape.

-----------------------------------------------------------------

70 - Most common manifestation of cardiac lupus is

a. Myocarditis
b. Libman Sachs endocarditis
c. Pericarditis
d. Aortic regurgitation

---------------------------------------------------------------------

The correct answer is C


Pericarditis, which is an inflammation of the membrane enveloping the heart, is the most frequent manifestation of cardiac lupus. The inflammation can cause the build up of fluid into the pericardial sac surrounding the heart and cause compression of the heart.

--------------------------------------------------------------------

71 - HLA - B27 histocompatibility antigen is correlated with

a. Sjorgen's disease
b. Ankylosing spondylitis
c. Felty's syndrome
d. Scleroderma

-----------------------------------------------------------------------

The correct answer is B


The spondyloarthropathies are a family of interrelated, but heterogeneous, conditions. There can be a marked overlap between the different conditions. All of these conditions are associated to a greater or lesser extent with the HLA B27 antigen. The diseases included in this group are ankylosing spondylitis, Reiter’s syndrome, reactive arthritis, psoriatic arthritis, and the enteropathic arthritis associated with inflammatory bowel disease.

Disease classified under the spondyloarthropathies

o Ankylosing spondylitis (AS)
o Reiter’s syndrome / Reactive arthritis
o Enteropathic arthritis (Crohn’s disease, ulcerative colitis)
o Psoriatic arthritis



Although the precise etiology is unknown, there is a strong association between this group of diseases and the HLA-B27 antigen. HLA-B27 forms part of the class I antigens of the major histocompatibility complex (MHC) and is expressed on the surface of many cell types. This supports the view that these diseases are due to a genetically determined immune response to environmental factors.

One hypothesis is that an immune response is triggered by an infection in the gastrointestinal or genitourinary tracts. Some Chlamydia, Salmonella and Yersinia antigens can be found in the synovial fluid of patients who had infections with these pathogens. Some peptide fragments from these bacteria possibly bind to the HLA-B27 antigen and are presented to the T-cells, thus triggering an autoimmune response.

An alternative hypothesis suggests an autoimmune reaction to self-antigens, because of mimicry between fragments of the infecting microorganisms and host tissue i.e. antigenic fragments between fragments of bacteria and self-antigens. These infecting organisms may be Gram negative bacteria, such as Klebsiella which have been shown to possess antigens that resemble the HLA-B27 antigen. They may give rise to antibodies that crossreact and bind to HLA-B27 positive cells resulting in disease manifestations.

-----------------------------------------------------------------------------------------------

72- Which of the following joint findings is most suggestive of an inflammatory, rather than an osteoarthritic, cause of joint pain?

a. Painful range of motion
b. Crepitus
c. Bony articular enlargement
d. Swelling and warmth

---------------------------------------------------------------------------------------------

The correct answer is D

--------------------------------------------------------------

73 - Maternal mortality rate in India is close to

a. 1 per 1000 live births
b. 3-4 per thousand live births
c. 8 per thousand live births
d. 10 per 1000 live births

---------------------------------------------------------------------------

The correct answer is B

For India, the National Family Health Survey of 1992-93 was the first to provide a national-level estimate of 437 maternal deaths per 100,000 births for the two-year period preceding the survey (International Institute for Population Sciences, 1995).

To fill the data gap, in recent times, the potential of the Sample Registration System - a dual record system for collecting data on births and deaths - for estimating maternal mortality has also been explored. The source has recorded a maternal mortality rate of 408 and 407 for 1997 and 1998, respectively (India, Registrar General, 1999 and 2000).

------------------------------------------------------------------------

74 - The earliest case of Severe Acute Respiratory Syndrome (SARS) was reported in

a. China
b. Singapore
c. Vietnam
d. Toronto

----------------------------------------------------------------------------

The correct answer is C

In March 2003, a Chinese-American businessman died in Vietnam from a severe flu-like illness. Soon after, reports of a new disease, then termed "atypical pneumonia," made headlines around the world.

The first cases of SARS probably arose in November 2002 in Foshan City, located in the Guangdong Province of China. The Chinese government did not publicize the outbreak right away, however. In February 2003, the Chinese Ministry of Health reported 305 cases, with five fatalities, of a disease that quickly deteriorated the patient's health and evolved to respiratory failure. About one third of those affected were health care providers.

SARS first gained attention outside of China in March 2003, when Dr. Carlo Urbani, a WHO official based in Vietnam, reported several cases of "atypical pneumonia" at the hospital where he worked. The first of these cases was the Chinese-American businessman. Following these reports, the WHO was finally able to trace the disease and account for its spread across so many countries. On March 29, Dr. Urbani himself died of SARS.

-------------------------------------------------------------------

75- The life expectancy in India as per 2001 censusis

a. 55 yrs
b. 63 yrs
c. 70 yrs
d. 75 yrs

--------------------------------------------------------------------------

The correct answer is B

According to the Population Reference Bureau's 2000 World Data Sheets, life expectancy at birth for Indians is between 60 and 61 years. This was also confirmed by the most recent Census of India in 2001. Only 4% of our population is over the age of 65.

Country: INDIA
Population: 1,065,462,000 (2003)
Capital City: New Delhi
National Language: Hindi
Per capita income: US$ 530/year (2003)
Country: INDIA
Life expectancy: 64 years (2003)
Access to safe water: 86% (2002)
Under 5 Mortality Rate: 87/1,000 live births (2003)

---------------------------------------------------------------------------------

76 - Cortical lesions are usually accompanied by word blindness due to involvement of

a. Angular gyrus
b. Lateral geniculate body
c. Occipital cortex
d. Edinger-Westphal Nucleus

--------------------------------------------------------------------------------

The correct answer is C

Alexia - pure word blindness

* results from damage to left occipital cortex and corpus callosum caused by a stroke.
* here the right visual field is lost because this projects onto the left visual cortex.

--------------------------------------------------------------------------------

77- Which of the following organs is the most common site of origin of the tumor associated with the Zollinger-Ellison syndrome?

a. Duodenum
b. Lymph nodes
c. Spleen
d. Gall stones

--------------------------------------------------------------------------

The correct answer is D

The gastrinoma syndrome, or Zollinger-Ellison syndrome, has traditionally been associated with a severe, fulminant ulcer diathesis, often with multiple ulcers, and ulcers in unusual locations such as the post-bulbar region of the duodenum and proximal jejunum.

However, with increasing recognition of this syndrome, as well as the availability of a radioimmunoassay for serum gastrin, most patient's with gastrinoma now present with either milder forms of peptic ulcer disease, or with secretory diarrhea.

The syndrome can exist in multiple forms, including benign sporadic, malignant metastatic, and as part of the MEN-I syndrome.

Approximately 66% of gastrinomas are sporadic. Sporadic tumors are reported to be malignant in approximately 40-85% of cases. Sporadic gastrinomas occur primarily in the gastrinoma triangle, defined as the confluence of the cystic and common bile duct superiorly, the second and third portions of the duodenum inferiorly, and the neck and body of the pancreas medially.

Although they can occur in the pancreas, the duodenum has been shown to be the most common site of gastrinomas, based on the pioneering work of Debas and colleagues, Thompson and colleagues, and others. Duodenal wall gastrinomas have been identified in 43-77% of patients.

-------------------------------------------------------------------------------

78 - Which of the following is the most common non-alcoholic cause of acute pancreatitis?

a. Thiazides
b. Hypercalcemia
c. Hyperlipidemia
d. Gall stones

--------------------------------------------------------------------

The correct answer is D

Alcohol is responsible for about 30% of all cases of acute pancreatitis. The amount of alcohol required to induce clinically acute pancreatitis is controversial. Sarles suggests that an average intake of 80g of alcohol needs to be taken daily for 5 to 15 years before 1st attack. Acute attacks may occur with lesser, intermittent consumption (ie weekend binging).

Gallstones are the leading cause of acute pancreatitis in most series (30-60%), however only 3-8% of patients with symptomatic cholethiasis will develop acute pancreatitis.

The cause of AP in about 1.3-3.8% of cases. Hyperlipidaemia types 1,4 or 5 cause pancreatitis; is generally associated with serum triglyceride levels > 1000mg/dl.

-------------------------------------------------------------------------

79 - To find out whether the water has been properly chlorinated, the test used is

a. Horrock's test
b. Ortho Toluidine test
c. Presumptive coliform test
d. Total bacterial count

----------------------------------------------------------------------------

The correct answer is B

Chlorination status of water is confirmed by carrying out Ortho Toluidine test where in 1 ml sample of drinking water a drop of Ortho Toluidine reagent is added and after 10 seconds the yellowish coloration is compared with the standard colored plates of choloscope for the presence of free chlorine.

-------------------------------------------------------------------------

80 - Mid-day meal contains

a. 1/2 of proteins and 1/2 of the calories required per day
b. 1/2 of proteins and 1/3 of the calories required per day
c. 1/3 of proteins and 1/3 of the calories required per day
d. 1/3 of proteins and 1/2 of the calories required per day

---------------------------------------------------------------------------

The correct answer is B

The National Programme of Nutritional Support for Primary Education (i.e. the national
“mid-day meal scheme”) was initiated in 1995. By 2001 a few states were providing cooked meals, but most were only giving monthly “dry rations” of foodgrain to school children. The number of states providing cooked meals rose sharply from early 2002 onwards, after a Supreme Court order (dated 28 November 2001) directed all State Governments to introduce cooked mid-day meals in primary schools.

The Programme guidelines have also been revised in September, 06 wherein the nutritional norms were raised from the existing 300 calories and 8-12 grams of proteins to minimum 450 calories and 12 grams. of proteins.

From the allowances recommended by the Indian Council of Medical Research (ICMR) for children in the age range 6 to 10 years, one third of the daily requirements are computed as the nutrients and calories to be supplied by the school lunch.

--------------------------------------------------------------------------------

81 - The non-modifiable risk factor for hypertension is

a. Obesity
b. Age
c. Salt intake
d. Environmental stress

--------------------------------------------------------------------------

The correct answer is B

--------------------------------------------------------------------------

82 - The Aqueous flare is best seen by

a. Streak retinoscope
b. Operating microscope
c. Biomicroscope
d. Ophthalmoscope

----------------------------------------------------------------------

The correct answer is D

Aqueous flare - The release of protein following breakdown of the blood-aqueous barrier renders the aqueous no longer optically clear. The path of a light shone across the anterior chamber is defined in the aqueous. A sharp focussed beam and careful observation are needed. Aqueous flare is usually transient.

To see aqueous flare an intense and focal light source must be held close (1 cm) to the corneal surface. A direct ophthalmoscope set at its smallest circular aperture of white light, and to its highest light intensity, works best (a pen light is inadequate for this procedure). Aqueous flare is present if the light beam is visible in the anterior chamber.

--------------------------------------------------------------------

83 - 'Malignant Otitis Externa' is caused by

a. Rhabdomyosarcoma
b. Squamous cell carcinoma
c. Stewarts granuloma
d. Pseudomonas Aeruginosa

----------------------------------------------------------------

The correct answer is D


Pseudomonas aeruginosa is the predominant pathogen associated with otitis externa. Occasionally P. aeruginosa may penetrate the epithelium of the floor of the external auditory canal & invade the underlying soft tissue, cartilage & cortical bone. This condition is called malignant otitis externa & occurs predominately in elderly diabetics. Virtually all cases of malignant otitis externa are caused by P. aeruginosa.

-----------------------------------------------------------------

84 - If death of a patient occurs during surgery due to the negligence of the surgeon, then he can be charged under

a. 299 IPC
b. 300 IPC
c. 304 (A) IPC
d. 304 (B) IPC

-------------------------------------------------------------------

The correct answer is C

Gross negligence recognised as 'criminal negligence' is deemed as an offence, though criminal intention may not be present. Section 304-A of IPC deals with causing death by any rash or negligent act and prescribes a punishment of imprisonment of either description for a term which may extend to two years or with fine or with both.

-------------------------------------------------------------------------

85 - Which of the following poisons has only local actions?

a. Sulphuric acid
b. Carbolic acid
c. Oxalic acid
d. Phosphorus

--------------------------------------------------------------

The correct answer is A

Poisons may exert a twofold action. This may be either local, or remote, or both local and remote.

The local action of a poison is usually one of corrosion, inflammation, or a direct effect upon the sensory or motor nerves.

The remote actions of poisons are usually of a specific character, though some writers group the remote effects of poisons under two heads, and speak of the common and the specific remote effects of a poison.. The local action of a poison of the corrosive class is usually so well marked and obvious that the fact of the administration of a poison of this class is generally unmistakable.

The symptoms produced by the mineral acids and the alkalies are almost altogether referable to local action; but some corrosive poisons, such as carbolic acid, produce, besides a local action, remote and specific constitutional effects.

----------------------------------------------------------------------------


86 - Copper is a constituent of the enzyme

a. Lysyl oxidase
b. Glucose oxidase
c. Xanthine oxidase
d. Transketolase

--------------------------------------------------------

The correct answer is A


Copper is a cofactor for lysyl oxidase, the enzyme that forms cross-links between collagen in connective tissue.

-------------------------------------------------------

87 - The principle that in a spinal cord dorsal roots are sensory and the ventral roots are motor is known as

a. Laplace's Law
b. Bell-Magendie's Law
c. Frank- Starling's Law
d. Weber-Fechner's Law

-------------------------------------------------------------

The correct answer is B

The anterior spinal nerve roots contain only motor fibres and posterior roots only sensory fibres.

Charles Bell's work of 1811 Contains the first reference to experimental work on the motor functions the ventral spinal nerve without, however, establishing the sensory functions of the dorsal roots.

In 1822 Magendie definitively discovered that the anterior root is motor and that the dorsal root is sensory. Magendie announced that "section of the dorsal root abolishes sensation, section of ventral roots abolishes motor activity, and section of both roots abolishes both sensation and motor activity". This discovery has been called "the most momentous single discovery in physiology after Harvey". In the same volume of Journal de physiologie expérimentale et de pathologie, Magendie gave experimental proof of the Bell-Magendie law.

Magendie proved Bell’s law by severing the anterior and posterior roots of spinal nerves in a litter of puppies. Stimulation of the posterior roots caused pain.

-----------------------------------------------------

88 - Immunity acquired due to injection of immunologically competent lymphocytes is termed as

a. Innate immunity
b. Adoptive immunity
c. Active immunity
d. Local immunity

---------------------------------------------------------------

The correct answer is B

Antibody-mediated immunity is active when the person synthesizes his own antibody; it is passive when another person or animal synthesizes the antibody and it is subsequently injected into the person receiving the passive immunization.

Cell-mediated immunity is active when the person's own T-cells are stimulated or adoptive when the source of stimulated T-cells is another person.

Adoptive immunization is rarely used with humans because the recipient will reject the cells unless they are histocompatible (genetically identical, e.g. from an identical twin).

Innate immunity is present from birth and is independent of the life experiences of the person, whereas acquired immunity arises only after an infection or immunization and hence is acquired during life.

---------------------------------------------------------------------------

89 - Which cholesterol is designated "good Cholesterol"?

a. VLDL
b. LDL
c. HDL
d. IDL

-----------------------------------------------------------------

The correct answer is C


CHOLESTEROL: Cholesterol is a fat-like substance found in all foods of animal origin (meat, poultry, fish, and dairy products) but not in food from plants. It is also produced by the liver. Cholesterol is a normal component of the blood and is needed for nerves, cell walls, and hormones. However, too much cholesterol can build up in arteries leading to heart disease, stroke, hypertension, and atherosclerosis. Cholesterol is transported in the blood in several forms - two important forms are high-density lipoprotein (HDL) and low-density lipoprotein (LDL). Levels are dependent upon age, diet, heredity and exercise.

LIPOPROTEIN: A combination of a lipid (fat) and a protein. Because fat and water don’t mix, cholesterol must be carried in the blood by protein capsules.

LDL CHOLESTEROL (Low Density Lipoprotein): This is the form of cholesterol often termed “the bad cholesterol.” It tends to build up on blood vessel walls, increasing the risk of heart disease.

HDL CHOLESTEROL (High Density Lipoprotein): HDL cholesterol, often termed “the good cholesterol,” appears to remove cholesterol from the body. Large amounts of HDL cholesterol are associated with a decreased risk of heart disease.

TOTAL CHOLESTEROL/ HDL RATIO: This value is used to predict heart disease risk.

TRIGLYCERIDE: The form of fat in the body's fat deposits. It forms when we eat more than we burn off. Triglycerides are a source of stored energy and can be imported through the blood. The role of high blood triglycerides in heart disease is unclear. However, for some people high triglycerides are a risk factor similar to high cholesterol. In this test, it is used to calculate LDL levels, as LDL values are difficult to measure directly.

----------------------------------------------------------------

90 - Which of the following is the most specific test to diagnose syphilis?

a. VDRL
b. Wassermann
c. RPR
d. FTA-ABS

-----------------------------------------------------------------

The correct answer is D


FTA-ABS stands for fluorescent treponemal antibody absorbed, a blood serum screening test for syphilis designed to demonstrate the presence or absence of specific antibodies directed against the organism (Treponema pallidum) responsible for syphilis.

----------------------------------------------------------------------

91 - Following is a clinical feature of cerebellar disease

a. Paralysis
b. Sensory deficit
c. Ataxia
d. Resting tremors

------------------------------------------------------

The correct answer is C

The cardinal clinical features of cerebellar disease are ataxia, dyssynergia, and dysmetria. Ataxia of gait is typified by unsteadiness with a wide base, body sway, and an inability to walk on tandem (heel to toe). Dyssynergia refers to a decomposition of movement instead of a smooth, continuous movement; it is associated with a tendency to miss a target and worsens when approaching the target. Dyssynergia is frequently accompanied by dysmetria (the misjudging of distance), with its characteristic overshooting and undershooting of a target.

-----------------------------------------------------------

92 - Oxytocin induced myometrial contractions are inhibited by all of the following EXCEPT

a. Ergometrine
b. Terbutaline
c. Magnesium
d. Halothane

-----------------------------------------------------------
The correct answer is A

Ergometrine is an amine ergot alkaloid that stimulates contractions of uterine and vascular smooth muscle.

-----------------------------------------------------------

93 - Effective for common gram negative anaerobes

a. Aztreonam
b. Doxycycline
c. Vancomycin
d. Tobramycin

-----------------------------------------------------------

The correct answer is A

Aztreonam (Azactam)

* Synthetic
* Primarily active against aerobic gram-negative bacteria (E. coli, Klebsiella, Pseudomonas)
* Bactericidal
* Used for severe systemic infections and UTIs

------------------------------------------------------------------

94 - Stein-Leventhal syndrome is

a. Pelvic endometriosis
b. Polycycsitc ovarian disease
c. Functioning granulosa cell ovarian tumor
d. Endometrial hyperplasia

-------------------------------------------------------------------------

The correct answer is B

* Polycystic ovary syndrome (PCOS) or polycystic ovary disease (PCOD), the association of multicystic ovaries with three cardinal physical manifestations, irregular or absent menstrual periods, increased body hair growth (hirsutism), and obesity, has been recognized at least since the 1930’s.
* The irregular or absent menstrual periods are due to infrequent or absent ovulation, which often causes the additional symptom of infertility.
* Drs. Stein and Leventhal were among the first physicians to describe and investigate the syndrome; thus PCOS/PCOD is also known as Stein-Leventhal Syndrome.

----------------------------------------------------------

95 - Which of the following anemias is a risk factor for the development of Gastric cancer?

a. Pernicious anemia
b. Megaloblastic anemia
c. Aplastic anemia
d. Hemolytic anemia

---------------------------------------------------------------

The correct answer is A

Pernicious anemia follows autoimmune gastritis. This is a disease common in individuals from northern Europe, particularly Scandinavia. It is characterized by a severe atrophy of the gastric body mucosa with accompanying intestinal metaplasia. Because there is destruction of the glands patients develop achlorhydria and an absence of intrinsic factor secretion.

------------------------------------------------------------------------------

96 - All are features of irritable bowel syndrome EXCEPT

a. Vomiting
b. Pain abdomen
c. Diarrhea
d. Constipation

---------------------------------------------------------------

The correct answer is A


IBS is not a serious health threat and it doesn’t lead to any significant complications, but it can be a major nuisance. It is the most common disorder of the digestive tract and causes a pattern of discomfort and distress. Symptoms include recurrent attacks of bloating, belching or gas, lower abdominal cramps, diarrhea, constipation or alternating bouts of diarrhea and constipation.

------------------------------------------------------------------

97 - The minimum amount of normal perfused liver parenchyma necessary to be left intact when a hepatic resection is planned is

a. 10%
b. 20%
c. 50%
d. 75%

-----------------------------------------------------------------

The correct answer is B


Hepatic resection is appropriate treatment for a variety of benign and primary or secondary malignant hepatic lesions. In appropriate patients with malignant disease, resection generally offers the best opportunity for long-term survival or cure. In patients with benign tumors, resection is safe and effective when carried out for symptoms directly related to the tumor.

In a healthy,noncirrhotic patient, up to 80 percent of the functional parenchyma can be removed with a reasonable expectation of subsequent regeneration.

--------------------------------------------------------------

98 - Senile Vaginitis is due to

a. Gonococcal infection
b. Cancer cefvix
c. Diabetes
d. Oestrogen deficiency

----------------------------------------------------------------------

The correct answer is D

Senile or atrophic vaginitis is an inflammation or irritation of the vagina caused by thinning and shrinking of the tissues of the vagina and decreased lubrication of the vaginal walls. This is due to a lack of oestrogen. This condition is common in post menopausal women.

--------------------------------------------------------------------------

99 - In a highly selective vagotomy, the vagal supply is severed to

a. Proximal two-thirds of stomach
b. Antrum
c. Pylorus
d. Whole of stomach

----------------------------------------------------------------------

The correct answer is A

Unlike other forms of vagotomy, this version only disables the nerve fibers which control acid-secreting glands, while leaving other vagus nerve functions undisturbed.

Highly selective vagotomy (HSV), also referrred to as parietal cell vagotomy or proximal gastric vagotomy, divides only the vagus nerves supplying the proximal two-thirds of stomach.

-------------------------------------------------------------------------

100 - A patient with Ca. Cheek has tumor of 2.5 cms located close to and involving the lower alveolus. A single mobile homolateral node measuring 6 cms is palpable. Based on these clinical findings TNM stage of tumor is

a. T1N1M0
b. T2N2M0
c. T3N1M0
d. T4N2M0

------------------------------------------------------------------------------------------------------

The correct answer is D

TNM Staging of Oral Cancer

Primary Tumor (T)
TX Primary tumor cannot be assessed
T0 No evidence of primary tumor
Tis Carcinoma in situ
T1 Tumor 2 cm or less in greatest dimension
T2 dimension Tumor more than 2 cm but not more than 4 cm in greatest
T3 Tumor more than 4 cm in greatest dimension
T4 Tumor invades adjacent structures (e.g., through cortical bone, into maxillary sinus, skin, pterygoid muscle, deep muscle of tongue)
Nodal Involvement (N)
NX Regional lymph nodes cannot be assessed
N0 No regional lymph node metastasis
N1 Metastasis in a single ipsilateral lymph node, 3 cm or less in greatest dimension
N2 Metastasis in a single ipsilateral lymph node, more than 3 cm but not more than 6 cm in greatest dimension; or in multiple ipsilateral lymph nodes, none more than 6 cm in greatest dimension; or in bilateral or contralateral lymph nodes, none more than 6 cm in greatest dimension
N2a Metastasis in a single ipsilateral lymph node, more than 3 cm but not more than 6 cm in greatest dimension
N2b Metastasis in multiple ipsilateral lymph nodes, none more than 6 cm in greatest dimension
N2c Metastasis in bilateral or contralateral lymph nodes, none more than 6 cm in greatest dimension
N3 Metastasis in a lymph node more than 6 cm in greatest dimension
Distant Metastasis (M)
MX Distant metastasis cannot be assessed
M0 No distant metastasis
M1 Distant metastasis
Stage Grouping
Stage 0 Tis N0 M0
Stage I T1 N0 M0
Stage II T2 N0 M0
Stage III T3 N0 M0; T1 or T2 or T3 N1 M0
Stage IV Any T4 lesion, or

Any N2 or N3 lesions, or

Any M1 lesion

---------------------------------------------------------------------------

101 - Pneumatoceles often develop in children after pneumonia due to the following organism

a. Klebsiella
b. Streptococcus
c. Staphylococcus Aureus
d. Hemophillus Influenza

-----------------------------------------------------

The correct answer is C


Pulmonary pneumatoceles are thin-walled, air-filled cysts that develop within the lung parenchyma. They can be single emphysematous lesions but more often are multiple, thin-walled, air-filled, cystlike cavities. Most often, they occur as a sequela to acute pneumonia, commonly caused by Staphylococcus aureus. Incidence of postinfectious pneumatocele formation ranges from 2-8% of all cases of pneumonia in children. However, the frequency can be as high as 85% in staphylococcal pneumonias.

-------------------------------------------------------

102 - Which of the following mycobacteria can cause disease in HIV +ve patient with a CD4 count of 600 / cu.mm?

a. M. tuberculosis
b. M. avium-intracellulare
c. M. chelonei
d. M. fortuitium

-----------------------------------------------------------

The correct answer is A

* Absolute CD4 count: normal value would be 600-1200 in an adult.
* CD4 percentage: normal value would be 30-60%
* CD8 count: normal value would be 420-660 in an adult.
* CD4:CD8 ratio: normal value would be 1.2-1.9.

* CD4 count less than 200 is AIDS defining.
* CD4 percentage of less than 20% indicates serious immunodeficiency and a value of less than 14% indicates AIDS.

Mycobacterium avium complex (MAC) consists of two species—M avium and Mycobacterium intracellulare. MAC causes disseminated infection in immunocompromised hosts, but only a minority of immunocompetent hosts develop MAC lung disease. Patients with AIDS and lymphomas usually develop disseminated MAC (DMAC) infection when their CD4 count falls to less than 50 cells/mL.

--------------------------------------------------------------

103 - The most common vertebral lesion seen in spinal tuberculosis is

a. Central cystic type
b. Anterior type
c. Paradiscal
d. Posterior spinal disease

-------------------------------------------------------------

The correct answer is B

The disease begins in the vertebral bodies anteriorly causing destruction, involvement of the disc and slow collapse of the spine. The deformity is angular in the A.P. Plane.

Vertebral body destruction or 2 adjacent vertebral bodies with narrowing of the disc between. In the early stages with destruction more anterior.

As the disease progresses there will be PARAVERTEBRAL ABSCESS formation.

----------------------------------------------------------------

104 - A person who is very rigid, inflexible and loves order and discipline, can be categorized as

a. Hysterical
b. Anankastic
c. Affective
d. Explosive

---------------------------------------------------------------------

The correct answer is B

Anankastic (Obsessive-Compulsive) Personality Disorder

Personality disorder characterized by at least 3 of the following:

1. feelings of excessive doubt and caution;
2. preoccupation with details, rules, lists, order, organization or schedule;
3. perfectionism that interferes with task completion;
4. excessive conscientiousness, scrupulousness, and undue preoccupation with productivity to the exclusion of pleasure and interpersonal relationships;
5. excessive pedantry and adherence to social conventions;
6. rigidity and stubbornness;
7. unreasonable insistence by the patient that others submit to exactly his or her way of doing things, or unreasonable reluctance to allow others to do things;
8. intrusion of insistent and unwelcome thoughts or impulses.

Includes:

* compulsive and obsessional personality (disorder)
* obsessive-compulsive personality disorder

Excludes:

* obsessive-compulsive disorder

---------------------------------------------------------------------------------

105 - Mineralisation of the teeth begins at

a. Crown and progresses towards root
b. Root and progresses towards crown
c. simultaneously at root and crown
d. Begins in the centre

-------------------------------------------------------------------

The correct answer is A

Teeth form mainly from neuroectoderm and comprise a crown of insensitive enamel surrounding sensitive dentine and a root that has no enamel covering. Tooth eruption occurs after formation and mineralization of the crown are largely complete but before the roots are fully formed.

The teeth calcify from crown to neck to roots.

-------------------------------------------------------------------

106 - Early and reliable indication of air embolism during anesthesia can be obtained by continuos monitoring of

a. ECG
b. Blood pressure
c. End tidal CO2
d. Oxygen sturation

----------------------------------------------------------------------

the correct answer is C

Venous air embolism may occur during any operative procedure in which the operative site is above the level of the heart and non-collapsible veins are exposed to atmospheric pressure, or when air or any other gas is introduced under pressure into a body cavity.

Determination of end-tidal CO2 (ETCO2) concentration to confirm adequate ventilation is useful during all anesthetic procedures, but particularly so for general anesthesia. A rapid fall of ETCO2 is a sensitive indicator of air embolism, a major complication of sitting craniotomies.

-----------------------------------------------------------------------

107 - Which of the following nerve lies closest to the Wharton's duct?

a. Hypoglossal
b. Lingual
c. Chorda Tympani
d. Facial

--------------------------------------------------------------------------

The correct answer is B

The sublingual gland lies against the sublingual depression of the mandible and directly on the mylohyoid. The submandibular duct (Wharton duct) and the lingual nerve lie medial to the gland.

---------------------------------------------------

108 - Thyroid carcinoma with pulsating vascular skeletal metastasis is

a. Follicular
b. Anaplastic
c. Medullary
d. Papillary

----------------------------------------------------

The correct answer is A

--------------------------------------------------

109 - The normal narrowing in the middle of the esophagus is due to

a. Azygous vein
b. Hemiazygos vein
c. Right main stem bronchus
d. Left main stem bronchus

---------------------------------------------------

The correct answer is D

The esophagus, in its course through the mediastinum, has several narrowings. The first is the upper esophageal sphincter and the next is the region of aortic crossing where the transition from striated to smooth muscle is usually found. Narrowed areas can be found where the left mainstem bronchus crosses and where the left atrium lies next to the esophagus. The last narrowing is in the zone of the lower esophageal sphincter.

----------------------------------------------

110 - Chronic treatment with Tamoxifen can cause carcinoma of

a. Ovary
b. Endometrium
c. Cervix
d. Vulva

--------------------------------------------------------------

The correct answer is B

Tamoxifen may cause changes in the lining of the uterus. An early sign of these changes may be abnormal vaginal bleeding or pelvic pain. After an average of 8 years of follow-up, the annual (per year) risk observed in a large-scale trial of breast cancer patients taking 20 mg of tamoxifen daily is about 2 per 1,000 women. This means that on the average, two cases of endometrial cancer were diagnosed among every 1000 women receiving tamoxifen during each year of the study and follow-up. This level of risk is approximately three times greater than that of a similar group of women in the general population.

------------------------------------------------------------

111- Puerperal Pyrexia is defined as

a. Temparature of 99F within 24hrs of delivery
b. Temparature of 99F 24hrs after delivery
c. Temparature of 100.4F 24hrs after delivery
d. Temparature of 102F after 45 days of delivery

---------------------------------------------------------

The correct answer is C

Puerperal pyrexia is considered as any febrile condition occurring in a woman whose temperature rises to 100.4’ F (38 C) or more within fourteen days after confinement or miscarriage.

-----------------------------------------------------------

112 - Maternal mortality ratio is the number of maternal deaths that result from the reproductive process

a. per 1,00,000 live births
b. per 1,00,000 total births
c. per 10,000 live births
d. per 10,000 total births

-------------------------------------------------------------

The correct answer is A

Levels of maternal mortality are measured by the maternal mortality ratio, or the number of maternal deaths per 100,000 live births. Progress towards the reduction of maternal mortality is assessed by tracking the availability of professional care at delivery, on the basis of process indicators such as the percentage of deliveries assisted by a skilled attendant.

------------------------------------------------------------

113 - MRI is the investigation of choice in all of the following EXCEPT

a. Syringomyelia
b. Brain stem tumors
c. Skull bone tumors
d. Multiple Sclerosis

----------------------------------------------------

The correct answer is B ( check )

MRI is investigation of choice in soft tissue pathology , whereas for bone imaging CT is the investigation of choice.

--------------------------------------------------------------

114 - Crohn's disease can be seen in

a. Jejunum only
b. Colon only
c. Terminal ileum and right side of colon
d. Mouth to anus

--------------------------------------------------

The correct answer is D

* Crohn's disease is a systemic disorder whose principal clinicopathological manifestation is chronic inflammation of the intestine.
* Any part of the gastrointestinal tract from mouth to anus may be involved in the chronic granulomatous process, but the terminal ileum and colon are the regions most frequently affected.
* The disease commonly presents with abdominal pain, loss of energy and loss of weight, night sweats, mouth ulcers and joint pains.
* About 60% of people have diarrhoea which in Crohn's colitis contains mucous, pus and blood.
* The tissues around the anus and perineum may be chronically inflamed discharging pus from multiple sinuses.

-------------------------------------------------------------------------

115 - Bronchiectasis is associated with which syndrome?

a. Alpha 1 antitrypsin deficiency
b. Kartagener syndrome
c. Steven-Johnson syndrome
d. Stickler syndrome

----------------------------------------------------------

The correct answer is B

* Bronchiectasis refers to a dilation of the bronchi due to inflammatory changes with accumulation of secretions primarily affecting the bronchial wall.
* Rarely, bronchiectasis can be congenital, but it is usually secondary to chronic pulmonary infections (most commonly CF).
* Bronchiectasis may also be seen in the setting of GER. Bronchiectasis is characterized by a chronic productive cough with repeat episodes of pneumonia involving the same lung segment (often the left lower lobe).
* Hemoptysis occurs in 50% of patients, and one may also see clubbing.
* Of note, bronchiectasis is one of the components of Kartagener Syndrome (also situs inversus, otitis, and chronic sinusitis).

------------------------------------------------------------

116 - The triad of Diabetes, Gall stones and steatorrhea is associated with which one of the following tumors?

A. Glucagonomas
B. Somatostatinoma
C. VIPomas
D. Gastrinoma

---------------------------------------------------------

the correct answer is B

-----------------------------------------------------------

117 - Validity of an Indicator means

a. It should actually measure what it is supposed to measure
b. The answer should be same if measured by different persons in similar circumstances
c. Indiactors should reflect changes in the situation concerned
d. Indicators should have ability to obtain data needed

---------------------------------------------------------------------------

The correct answer is A

Reliability refers to the accuracy and consistency of an indicator. An indicator is reliable if multiple uses of the same tool such as an interview or a survey yield the same or similar results. Validity of an indicator means that the information that the indicators provide must be close to the reality they are measuring. A valid indicator measures the intended concept and not something else.

-------------------------------------------------------------------

118 - The Millenium Development Goals (MDG) is to be achieved by the year

a. 2010
b. 2015
c.2020
d. 2030

----------------------------------------

The correct answer is B

The Millennium Development Goals (MDGs) are eight goals to be achieved by 2015 that respond to the world's main development challenges. The MDGs are drawn from the actions and targets contained in the Millennium Declaration that was adopted by 189 nations-and signed by 147 heads of state and governments during the UN Millennium Summit in September 2000.

-------------------------------------------------------

119 - The number of new cases occuring in a defined population during a specified period of time is called

a. Period prevalence
b. Point prevalence
c. Prevalence
d. Incidence

-----------------------------------------------

The correct answer is D

Incidence is the number of new events arising within a specified time period. The word incidence usually denotes incidence rate.

Prevalence is the number of events in a population at a designated time. The word prevalence usually denotes point prevalence ratio.

Point prevalence is the number of persons with a disease or attribute at a point in time.

Period prevalence is the number of persons who have had the disease or attribute at any time over a specified period.

----------------------------------------------------------------

120 - Disbursement of old age pension by the government is

a. Social assistance
b. Social welfare
c. Social insurance
d. Social defense

-------------------------------------------------------------------------

The correct answer is B

The National Social Assistance Programme (NSAP) is a social assistance programme for poor households and represents a significant step towards the fulfillment of the Directive principles in Articles 41 and 32 of the Constitution recognizing the concurrent responsibility of the Central and State Governments in the matter.

NSAP was included in the Central Budget for 1995-96. The NSAP will includes the following benefits as its components

(i) National Old Age Pension Scheme
(ii) National Family Benefit Scheme
(iii) National Maternity Benefit Scheme

The programme came into effect on 15th August 1995.

The scales of the benefit under NSAP is as below

* National Old Age Pension Scheme Rs. 75/- per month
* National Family Benefit Scheme Rs. 10,000/- in case of death of the primary breadwinner to the bereaved house hold.
* National Maternity Benefit Scheme - Rs. 500/- per pregnancy upto two live births

-------------------------------------------------------------------------------

121 - A primary deviation less than the secondray deviation is seen in

a. Concomitant squint
b. Paralytic squint
c. Spastic squint
d. Acute squint

------------------------------------------------

The correct answer is B


PARALYTIC SQUINT

* False orientation of the object: object is projected too far in the direction of paralyzed muscle, due to increase in secondary deviation.
* Vertigo and nausea: They are partly due to diplopia, and partly due to false orientation.
* Secondary angle of deviation is more than the primary deviation.
* Restriction of ocular movements in the direction of action of paralyzed muscle.
* Compensatory head posture:
* In paralytic squint to neutralize diplopia, the chin may be elevated or depressed.
* The face turned to right or left side.

------------------------------------------------------

122 - In Marfan's syndrome lens dislocation is commonly

a. Upwards
b. Downwards
c. Supero-temporally
d. Nasally

---------------------------------------------------

The correct answer is A

Many children with Marfan’s syndrome are born with a displaced lens in one or both eyes. It is usually displaced upwards. Lens dislocation may or may not be readily apparent. The displacement is usually not progressive but may contribute to the formation of cataracts.

Marfan Syndrome

Pathology;

Autosomal Dominant, disorder of alpha 1 unit of collagen synthesis

Clinical findings;

Arachnodactyly, pectus deformities, scoliosis, spondylolisthesis, heartvalve abnormalities, superior lens dislocation. Joint laxity, protrusio acetabuli

Management;
Joint laxity, conservatively, scoliosis and spondo. aggressively with surgery.Protrusio acetabuli with triradiate cartilage fusion

----------------------------------------------------------------------------

123 - Sympathetic Ophthalmia is characterized by

a. Lisch nodules
b. Busacca nodules
c. Koeppe nodules
d. Dalen Fuch's nodules

----------------------------------------------------------

The correct answer is D

Sympathetic Ophthalmia (SO):

* Rare, bilateral, immune-mediated, mild to severe granulomatous uveitis seen 2 weeks to 3 months (80%) after penetrating trauma or surgery.
* Scattered, multifocal, yellow-white subretinal infiltrations (Dalen-Fuchs’ nodules, 50%) with overlying serous retinal detachments, vitritis, and papillitis.
* Associated with inflammation in sympathizing (fellow) eye and worsened inflammation in exciting (injured) eye (keratic precipitates are an ominous sign);
* may have meningeal signs, poliosis, and alopecia (as in Vogt-Koyanagi-Harada syndrome).
* Patients have transient obscuration of vision, photophobia, pain, and blurred vision.
* Male predilection (probably reflects increased incidence of trauma in this group);
* chronic, recurring course;
* prognosis good (65% achieve >20/60 vision after treatment);
* associated with HLA-A11.

---------------------------------------------------------------------------------


124 - a mentally ill person smells pleasant or unpleasant odor whe none is present. This is an example of

a. Illusion
b. Delusion
c. Obsession
d. Hallucination

--------------------------------------------

The correct answer is D


Hallucinations are the persons’ perception of something that does not really exist in their environment.Hallucinations can affect most of the senses – the person may ‘see’ things (visual hallucinations); ‘smell’ things (olfactory hallucinations); or ‘feel’ things (tactile hallucinations).

----------------------------------------------------------

125 - With reference to an incised wound, all the following are true EXCEPT:

a. It has clean cut margins
b. Bleeding is generally less than in lacerations
c. Tailing is often present
d. Length of injury doesnot correspond with length of blade

----------------------------------------------------------

The correct answer is B


INCISED WOUNDS (CUTS)

Breach of the full thickness of the skin due to contact with a sharp edge

Forensic Importance

1. Reflects sharp edge not weapon type
2. No trace evidence
3. Bleeds profusely
4. Haemorrhage and air embolism

Direction of cut: deeper at the start (i.e tailing)

Suicidal or Homicidal?

* Site, e.g. neck, wrist, face
* Tentative wounds - suicide
* Defence wounds - homicide.

---------------------------------------------------------------

126 - Neutrophilic leucocytosis is seen in all EXCEPT:

a. Sepsis
b. Myocardial Infarction
c. Pyogenic osteomyelitis
d. Hay fever

-------------------------------------------------------

The correct answer is D

Neutrophilic leukocytes

* leukocytes are the first cells to appear at the site of acute inflammation
* is to degrade cell debris and to ingest and kill microbes- phagocytosis
* remain predominant cell type for several days in acute inflammation.

Eosinophils are the predominant inflammatory cells in allergic reactions and parasitic infestations. The most important causes of eosinophilia include allergies such as asthma, hay fever, and hives and also parasitic infections. Other causes include polyarteritis nodosa and Hodgkin lymphoma.

-----------------------------------------------------

127 - Munro microabcesses are characteristically seen in

a. Lichen planus
b. Lupus erythematosus
c. Psoriasis
d. Seborrheic dermatitis

-------------------------------------------------------

The correct answer is C

Psoriasis

Clinical:

* Well demarc red plaques, silvery scale, commonly located on knees elbow, scalp
* Auspitz sign: pinpoint bleeding, scale
* May be assoc with severe arthritis


Pathogenesis:

* Unknown but genetic factors play a role
* entire epidermis can express disease
* abnormal epidermal proliferation, transit time to surface reduced to 7 days from 28


Morphology:

* Confluent plaques, parakeratosis, hyperplasia
* PMNs in s.corneum, spongiform pustules, Munro microabcesses
* thin suprapapillary plate, tortuous capillaries

-----------------------------------------------------------------

128 - Cilastatain is an inhibitor of

a. Beta lactamase
b. Dehydropeptidase
c. Transpeptidase
d. Peptide Transferase

-------------------------------------------------------

The correct answer is B

The main drug in the carbapenam class is a drug called IMIPENEM – this drug has excellent, quite broad antimicrobial activity against both g+ and g- organisms, anaerobes, and Psuedomonas infections; it is given with a molecule called cilastatin, which is an inhibitor of the renal enzyme dehydropeptidase – this renal enzyme would otherwise break down the drug too quickly, and would make the drug ineffective in the treatment of UTI’s – thus by giving imipenem with the cilastatin, the half life of the drug is prolonged and it can be used to treat UTI’s.

-----------------------------------------------------------

129 - cAMP action mediates all EXCEPT

a. Glucagon
b. FSH
c. LH
d. Estrogen

-------------------------------------------------------

The correct answer is D

Some of the hormones that achieve their effects through cAMP as a second messenger:

* adrenaline
* glucagon
* luteinizing hormone (LH)
* Follicle stimulating Hormone (FSH)

-----------------------------------------------------------------

130 - In sickle cell anemia, the mutational event responsible for themutation of Beta chain is

a. Crossing over
b. Point mutation
c. Insertion
d. Deletion

----------------------------------------------------------

The correct answer is B

Point mutations = change of single nucleotide in the DNA.

Single nucleotide changes (point mutations) in the sequence of a gene can result in changes in the amino acid sequence of a protein produced from the mutated gene. One of the most well studied examples of the effects of a mutation on the sequence of a protein involves the oxygen-transporting protein hemoglobin. A point mutation creates an altered form of hemoglobin that produces the genetic disorder called sickle-cell disease (sickle-cell anemia).

Sickle-cell disease results from a point mutation in the second nucleotide of the codon GAA, which results in a change in the amino acid at position 6 in the hemoglobin protein.

----------------------------------------------------------------

131 - Surgical Neck of humerus is related to

a. Ulnar nerve
b. Radial Nerve
c. Median nerve
d. Axillary nerve

----------------------------------------------------------------

The correct answer is D

* The axillary nerve innervates teres minor and deltoid.
* It wraps around the surgical neck of the humerus and is endangered by fractures of the surgical neck.
* If the axillary nerve was damaged and deltoid was denervated, the patient would be unable to abduct his upper limb beyond 15 to 20 degrees.

-----------------------------------------------------------

132 - Which needle is used for pleural biopsy?

a. Vim silvermann's needle
b. Abram's needle
c. Abraham's
d. OsGood's

-------------------------------------------------------------------

The correct answer is B

The Abram’s pleural biopsy punch/needle, unlike some other pleural biopsy instruments, is a cutting needle. The outer needle has a cutting groove into which tissue either prolapses or is aspirated. The cutting trochar is then advanced, cutting the biopsy. The needle is then removed, reopened and the specimen taken out.

------------------------------------------------------------

133 - Which of Dens fractures is most common?

a. Type I
b. Type II
c. Type III
d. Type IV

----------------------------------------------------------------------

The correct answer is B

There are three types of dens fractures:

1. type 1- you will see a thin radiolucent above the base of the dens. This is a stable fracture.
2. type 2- you will see a thine radiolucent line at the base of the dens. This is the most common, most unstable, and the most severe.
3. type 3- you will se a thin radiolucent line under the base of the dens which arc upward. It is also stable.

----------------------------------------------------------------------

134 - Obsessive Compulsive disorder is distinguished from Psychosis by the absence of

a. Ritualistic behaviour
b. Perceptual anamolies
c. Anxiety
d. Violent behaviour

--------------------------------------------------------------

The correct answer is B


Someone with OCD may suffer from obsessions or compulsions or both. Obsessions are unwanted intrusive thoughts which repeatedly enters the person’s mind and compulsions which are repetitive behaviours or mental acts that the person is driven to perform. Compulsions can involve physical acts such as repeatedly checking that a door is locked or mental acts such as continually repeating a certain phrase in the mind.

Most individuals with OCD recognize at some point that their obsessions are coming from within their own minds and are not just excessive worries about real problems, and that the compulsions they perform are excessive or unreasonable.

----------------------------------------------------------------------

135 - Hypotension during anesthesia may be due to

a. Co2 retention
b. Isoflurane
c. Pancuronium
d. Vecuronium

---------------------------------------------------------------------

The correct answer is B

Blood pressure decreases during maintenance anaesthesia in relation to the depth
of anaesthesia. That is, blood pressure is inversely related to the ISOFLURANE
concentration. Provided there are no other complicating factors this is probably
due to peripheral vasodilation.

ISOFLURANE is a powerful systemic and coronary arterial dilator. The effect on
systemic arterial pressure is easily controlled in the normal healthy patient
and has been used specifically as a means of inducing hypotension. However, the
phenomenon of "coronary steal" means that ISOFLURANE should be used with caution in patients with coronary artery disease.

--------------------------------------------------------------

136 - Lucid Interval Is classically seen in

a. Intracerebral Hematoma
b. Acute subdural hematoma
c. Chronic Subdural Hematoma
d. Extradural hematoma

-------------------------------------------------------------------------

The correct answer is D

A lucid interval is especially indicative of an epidural hematoma. After the injury, the patient is momentarily dazed or knocked out, and then becomes relatively lucid for a period of time which can last minutes or hours. Thereafter there is rapid decline as the blood collects within the skull, causing a rise in intracranial pressure, which damages brain tissue.

----------------------------------------------------------------

137 - Non immune hydrops fetalis is caused by all EXCEPT

a. Parvo B19 virus
b. Chromosomal abnormalities
c. Alpha Thalassemia
d. ABO incompatibility

-----------------------------------------------------------------

The correct answer is D

Conditions associated with non-immune hydrops fetalis

Hemolytic anemia
-α-thalssemia
-RBC enzyme deficiencies

Other anemias
-Feto-maternal hemorrhage
-Twin-twin transfusion (donor)

Cardiac
-Fetal arrhythmias
-Premature closure of foramen ovale
-Hypoplastic left heart
-Hypoplastic right heart
-Ebstein’s anomaly of tricuspid valve
-Cardiomyopathy
-Cardiac tumors
-Premature closure of ductus
arteriosus
-Other structural anomalies

Chromosomal abnormalities
-Trisomy 21, 18
-Turner syndrome

Infections
-Viral (Parvovirus B19, Herpes,
CMV)
-Toxoplasmosis
-Syphilis
-Chagas Disease

Vascular malformations
-Chorioangioma (placenta, umbilical
vessels)
-Liver hemangioma
-Cerebral A-V malformation
-Sacrococcygeal teratoma
-Klippel-Trenaunay syndrome

Vascular accidents
-Intracranial hemorrhage
-Thrombosis of renal veins, IVC
-Twin-twin transfusion (recipient)

Lymphatic malformations
-Pulmonary lymphangiectasis
-Cystic hygroma
-Multiple pterygium syndrome
-Noonan syndrome

Chest masses
-Cystic adenomatoid malformation
-Diaphragmatic hernia
-Pulmonary sequestration
-Intrathoracic mass

Skeletal conditions
-Asphyxiating thoracic dystrophy
-Osteogenesis imperfecta
-Chondrodysplasia

Genetic metabolic disease
-Gaucher Disease
-Mucopolysaccharidosis
-Nieman-Pick Disease
-Neonatal hemochromatosis

Fetal Hypomobility
-Arthrogryposis
-Neu-Laxova syndrome
-Pena-Shokier syndrome
-Myotonic dystrophy

CNS anomalies
-Absent corpus callosum
-Encephalocele
-Holoprosencephaly

Other
-Bowel obstruction with perforation
(meconium peritonitis, volvulus)
-Infant of diabetic mother
-Prune belly syndrome
-Congenital nephrosis
-Maternal indomethacin therapy

----------------------------------------------------------------------

138 - Conn's Syndrome is Characterized by

a. Hyperinsulinism
b. Hyperthyroidism
c. Hypoadrenalism
d. Hyperaldosteronism

-----------------------------------------------------------------

The correct answer is D

Conn syndrome

* Primary hyperaldosteronism results from autonomous aldosterone secretion, which, in turn, leads to suppression of renin secretion.
* Primary aldosteronism usually occurs in individuals between the ages of 30 and 50 years and accounts for 1% of cases of hypertension.
* Primary hyperaldosteronism is usually associated with hypokalemia; however, more patients with Conn's syndrome are being diagnosed with normal potassium levels.
* Most cases result from a solitary functioning adrenal adenoma (approximately 70%) and idiopathic bilateral hyperplasia (30%).
* Adrenocortical carcinoma and glucocorticoid suppressible hyperaldosteronism are rare, each accounting for less than 1% of cases.
* Patients typically present with hypertension, which is longstanding, moderate to severe, and may be difficult to control despite multiple-drug therapy.
* Other symptoms include muscle weakness, polydipsia, polyuria, nocturia, headaches, and fatigue.
* Weakness and fatigue are related to the presence of hypokalemia.

-------------------------------------------------------------------------

139 - Respiratory Distress syndrome is due to defect in the biosynthesis of

a. Dipalmitoyl Lecithin
b. Dipalmitoyl Cephalin
c. Dipalmitoyl serine
d. Dipalmitoyl inositol

--------------------------------------------------------------------------

The correct answer is A


Surfactant in the lung

1. Secreted by Type II alveolar cells, surfactant lines the alveoli at the gas-liquid interface and has dipalmitoyl lecithin, (dipolmitoyl phosphotidyl choline=DPPC) as a major constituent.
2. Surfactant has 2 unique surface tension properties

* The average surface tension is low.
* Surface tension varies with area. Surface tension rises as area gets bigger and falls as area gets smaller.

Physiological importance of surfactant

1. Increases lung compliance because surface forces are reduced.

2. Promotes alveolar stability and prevents alveolar collapse. Decreased surface area lowers surface tension. Increased surface area increases surface tension. Small alveoli are prevented from getting smaller. Large alveoli are prevented from getting bigger.

3. Promotes dry alveoli. Alveolar collapse tends to “suck” fluid from pulmonary capillaries. Stabilizing alveoli (see b) prevents transudation of fluid by preventing collapse.

------------------------------------------------------------------------------

140 - The following are autosomal dominant disorders EXCEPT

a. Myotonic Dystrophy
b. von Willebrand Disease
c. Hemochromatosis
d. Marfan Syndrome

---------------------------------------------------------------------

The correct answer is C

Autosomal dominant disorders comprise the majority (about 75%) of known human Mendelian conditions. Because they do not 'skip generations' as a rule, the familial nature of these disorders is often apparent to members of an affected family, even before formal medical evaluation. Pedigrees typically have a vertical pattern, i.e. clear evidence of transmission from one generation to the next.

Hereditary hemochromatosis

* Inheritance: autosomal recessive
* Gene: HFE – codes for hepcidin
* Carrier frequency: 1 in 8 to 10 Caucasians
* Predicted incidence of disease therefore is 1 in 250-400
* BUT, penetrance of disease is very low 1-10% of patients homozygous for C282Y, That’s why no population screening
* Mutations: C282Y most important, H63D less so
* Assay approach: multiple (we use PCR/RFLP at UPMC)

----------------------------------------------------------------------------------

141 - Ewing's Sarcoma is common in

a. 1st decade
b. 2nd decade
c. 6th decade
d. 9th decade

--------------------------------------------------

The correct answer is B


Ewing’s Sarcoma:

* peripheral neuroepithelioma/PNET primitive neuroectodermal tumor
* Second decade
* Pelvis, femur, humerus
* Pain, fever, signs resembling infection
* Bone lesion – often diaphyseal, lytic, permeative; prominent periosteal reaction, soft tissue mass common.
* t(11;22) characteristic, less commonly t(7;22) or t(21;22).

------------------------------------------------------

142 - Age of gestation when ovaries and testis are first distinguishable

a. 4 weeks
b. 8 weeks
c. 12 weeks
d. 16 weeks

-------------------------------------------------

The correct answer is B

Testicular tissues, and in particular seminiferous tubules, are recognized in the human embryo at 7 weeks of fetal age (crown-rump length 13-20 mm).Orientation of the primordial gonad towards ovarian differentiation in XX subjects appears after the 2nd month of fetal age.

---------------------------------------------------

143 - Scrambled egg sign appearance is seen in:

a. Carcinoma Stomach
b. Carcinoma Gall Bladder
c. Pancreatic Carcinoma
d. Renal Carcinoma

-----------------------------------------------------------

The correct answer is B

-----------------------------------------------

144 - The most important use of Transrectal Ultrasonography is for

a. Screening for prostate cancer
b. Distinguishing prostate cancer from benign prostatic hyperplasia
c. Systemic prostate biopsy in suspected prostate cancer
d. Guiding transurethral resection of prostate cancer

----------------------------------------------------------------

The correct answer is C

Earlier studies concentrated on the ultrasonic appearances of prostate abnormalities such as benign prostatic hyperplasia (BPH), carcinoma of the prostate (CAP), prostatitis, prostatic abscess, and prostatic calculi. Since the introduction of the prostate-specific antigen (PSA) screening test and early detection of prostate cancer, the role of transrectal ultrasound (TRUS) has changed; it is mainly used to image the prostate and to aid in guided needle biopsy. Urologists have incorporated TRUS in their practices; moreover, TRUS also is widely used to deliver treatments such as brachytherapy and to monitor cryotherapy treatment for prostate cancer

------------------------------------------------------

145 - Which of the following diagnostic techniques is most specific for pulmonary thromboembolism?

a. Pulmonary angiography
b. Ventilation lung scanning
c. Perfusion lung scanning
d. Arterial blood gas analysis

-------------------------------------------------------------

The correct answer is A

Traditional evaluation of PE begins with the ventilation-perfusion (V/Q) scan, which is considered the pivotal diagnostic test. Unfortunately, the test results are often nondiagnostic.

Previously, most patients with low- or intermediate-probability V/Q scans and clinical suspicion for PE would undergo pulmonary angiography, which remains the gold standard for ruling out PE.

----------------------------------------------------------------

146 - The local anesthetic with the longest duration of action is

a. procaine
b. Bupivacaine
c. Lidocaine
d. Mepivacaine

------------------------------------------------------------

The correct answer is B

Generally, lidocaine is considered to have a rapid onset of action (10–15 minutes), while bupivacaine has an intermediate onset of action (20–30 minutes). Lidocaine has a moderate duration of action (60–120 minutes) while bupivacaine has the longest duration (three to eight hours).

---------------------------------------------------

147- Erb's point is formed by

a. C4 and C5
b. C5 and C6
c. C6 and C7
d. C8 and T1

---------------------------------------------------

The correct answr is B

Erb's point is the site in the lateral root of brachial plexus. It is formed by the union of C5& C6 roots, which later converges together.

----------------------------------------------------

148 - In alcoholic hepatitis, the specific enzyme raised is

a. Alanine Amino Transferase (ALT)
b. Aspartate Amino Transferase
c. Alkaline Phosphatase (ALP)
d. Gamma Glutamyl Transpeptidase (GGT)

---------------------------------------------------------

The correct answer is B

AST/ALT= Deritis ratio

* Usually is decreased with injury, since ALT rises more than AST.
o Injury include: viral hepatitis, toxic hepatic injury
* Exception: in alcoholic liver disease, the AST is > ALT so the Deritis ratio is increased.


A raised gamma glutamyltransferase is not confined to alcoholic liver disease as it can be induced by drugs (such as anticonvulsants, lipid lowering drugs, and oral contraceptives) and is also elevated in fatty liver disease. It is not always raised in chronic alcoholics, hence it is not sensitive for occult alcohol abuse.

The activity of alkaline phosphatase is increased up to 5-fold the upper limit of normal in situations where bile flow is obstructed. It is normal or only mildly elevated in hepatocellular damage.

-------------------------------------------------------------------------

149- All of the following drugs may be used in hypertensive crisis EXCEPT

a. Nifedepine
b. Labetalol
c. Furosemide
d. Pindolol

--------------------------------------------------------------

The correct answer is D

Antihypertensive Agents That Are Useful in Hypertensive Crises

* Nitroprusside (Nipride) IV
Initial dose 0.3 µg/kg/min (Max dose 10µg/kg/min) Onset of action <1>10 mg/dl is toxic; >20 mg/dl may be fatal) ; hypotension, N&V, apprehension, convulsion, twitching, psychosis, dizziness, etc. It has decreased efficacy in renal failure
* Nitroglycerin IV
Initial dose 5 µg/min (Max dose 100 µg/min) Onset of action <5>
* Labetalol (Trandate) IV
Mixed alpha/beta blocker, excellent for most hypertensive emergencies.
First or second line for eclampsia; excellent in catecholamine surges
Initial dose :20 mg IV bolus over 2 minutes, then 20- 80 mg q10 min prn, or Continuous infusion at 0.5-2.0 mg/min (Max dose 300 mg),
Onset of action <5>first-degree heart block, severe bradycardia
* Diazoxide (Hyperstat) IV
Initial dose: 1-3 mg/kg (up to 150 mg) IV bolus q5-15 min over 5 minutes until BP is controlled (Max dose 600 mg),
Onset of action <2>
* Nicardipine (Cardene) IV
Initial dose: 5 mg/h (Max dose 15 mg/h), Onset of action <1 color="#0000ff">Best use for Vascular surgery, subarachnoid hemorrhage
Side effects: Headache ,Elevated heart rate
Contraindication: Severe aortic stenosis
* Enalapril (Vasotec) IV
Highly variable response; precipitous BP drop in high-renin states, rarely angioedema, hyperkalemia, or acute renal failure.
Initial dose: 1.25 -5mg IV over 5 min q6h if needed, duration 6 hours
Best use for Left ventricular failure
Contraindication: Angioedema
* Diltiazem (Cardizem) IV
Initial dose 0.25 mg/kg over 2 min, followed by infusion of 0.35 mg/kg at an initial rate of 10 mg/hour
Onset: 3-30 min
Adverse effects: excessive hypotension, flushing
* Trimethaphan
Initial dose: 1 mg/min (Max dose 4 mg/min), Onset of action <5>
* Hydralazine (Apresoline) IV
Indicated primarily for eclampsia
Dose is 10-50 mg IV or IM titrate to effect (onset <20>
* Phentolamine
Mainly for catecholamine surges as in pheochromocytoma hypertension (pure alpha-adrenergic blockade)
Dose is 5-15mg IV; onset 1-2 minutes; duration 3-10 minutes
Side effects: Tachycardia, flushing and headache may occur
* Esmololol (Breviblock®)
Mainly for acute aortic dissection, perioperatively, acute coronary ischemia
May be used with caution in acute MI with depressed LV to modulate heart rate
Very short half life (2-4 minutes) non-selective ß-blockade
Dose is 250-500µg/kg/min for 1 minute, then 50-100µg/kg for 4 minutes
Sequence may be repeated, and continuous drip may be maintained
Onset of action is 1-2 minutes; 10-20 minute duration
Very close monitoring is required, and fluid load is large with this agent

-----------------------------------------------------------------

150 - Formication is associated with _____ abuse

a. Alcohol
b. Cocaine
c. Benzodiazepines
d. Analgesics

--------------------------------------------------------------------

The correct answer is B

Cocaine is called psychoactive because its actions modify the body’s normal synaptic transmission. Its euphoric high is the result of its action on the dopamine system (although its reinforcing qualities, while not yet completely understood, have recently been linked to serotonin as well).

The long-term effect is what is called “cocaine paranoia.” It is characterized by dysphoria, hallucinations, paranoid psychosis, extreme delusions, and irrational behavior. Also, many suffer from formication, or the hallucination that ants, insects, or snakes are crawling on or under the skin . Some people have actually bled to death trying to pick imaginary bugs out from under their skin, due to cocaine withdrawal .

----------------------------------------------------------------------------

151 -

----------------------------------------------------

152 - A 78 year old patient is anesthetized for resection of a liver tumor. After induction and tracheal intubation, a 20-guage arterial line is placed and is connected to a transducer which is located 20 cm below the leel of heart. The system is zeroed at the stopcock located at the wrist while the patient's arm is stretched out on an armboard. How will the arterial line pressure compare with thetrue blood pressure?

a. It will be 20 mmHg higher
b. It will be 15 mmHg higher
c. It will be the same
d. It will be 15 mmHg lower

-----------------------------------------------------------

The correct answer is B

Arm and body positions are also important for accuracy of readings. The standard position of the arm requires the forearm to be at heart level. If the arm is too high, blood pressure will be underestimated; if the arm is too low, which may occur with the subject simply in the supine position, blood pressure will be overestimated.

For converting values in centimeters of water to values in millimeters of mercury, the value given in centimeters of water is divided by the factor 1.36.

The arterial pressure varies in various locations, the more peripheral the arterial line, the higher the systolic and the lower the diastolic, but the main arterial pressure will remine the same. The pedal line will have a higher systolic than the femoral line.

---------------------------------------------------------------

153 - The ECG of a patient with an artificial pacemaker in the right ventricle shows

a. RBBB with a narrow QRS complex
b. RBBB with a broad QRS complex
c. LBBB with a broad QRS complex
d. LBBB with a narrow QRS comples

------------------------------------------------------------

The correct answer is C

A ventricular rhythm originating from a cardiac pacemaker is associated with wide QRS-complexes because the pacing electrode is (usually) located in the right ventricle and activation does not involve the conduction system. In pacer rhythm the ventricular contraction is usually preceded by a clearly visible pacer impulse spike. The pacer rhythm is usually set to 72/min..

----------------------------------------------------------------

154 - Magnesium level in blood increases in

a. Uncontrolled diabetes mellitus
b. Liver cirrhosis
c. Kidney failure
d. Chronic alcoholism

--------------------------------------------------

The correct answer is C

Poorly controlled diabetes increases loss of magnesium in urine and may increase an individual's need for magnesium.People who abuse alcohol are at high risk for magnesium deficiency because alcohol increases urinary excretion of magnesium.

Magnesium toxicity is more often associated with kidney failure, when the kidney loses the ability to remove excess magnesium.

--------------------------------------------------

155 - According to the concept of the Iceberg phenomenon of diseases, all of the following statements about the submerged portion of the ioceberg are true EXCEPT

a. It includes sub-clinical cases
b. It includes carriers
c. It constitutes undiagnosed reservoir of infection
d. Diagnosis can be made with available techniques

--------------------------------------------------------

The correct answer is D

The “iceberg of disease” is a metaphor emphasising that for virtually every health problem the number of cases of disease ascertained (those visible) is outweighed by those not discovered (those invisible).

Diseases may be mild or even ‘silent’ -one of the many explanations for undiagnosed disease in the community.This phenomenon is described by the metaphor of the iceberg of disease.

-------------------------------------------------------------

156 - Hemobilia is characterised by all EXCEPT

a. Jaundice
b. Biliary Colic
c. Malaena
d. Fever

------------------------------------------------------

The correct answer is D

Hemobilia: Hemorrhage in or through the biliary tract, due to trauma, inflammation, cholelithiasis, vascular disease, or neoplasms.Jaundice, right upper quadrant abdominal pain, hematemesis and melaena are the most frequent symptoms leading to admission.

------------------------------------------------------

157 - The period of training for a village health guide is

a. 1 month
b. 3 months
c. 6 months
d. 12 months

---------------------------------------------------

The correct answer is B


The training is arranged in the nearest subcenter, PHC or any other suitable place for the duration of 200hrs over a period of 3 months.

----------------------------------------------------------

158 - The type of laser employed for treating posterior capsule thickening is

a. Krypton
b. Excimer laser
c. Argon laser
d. YAG laser

-------------------------------------------------

The correct answer is D

SECONDARY CATARACT

* Clouding of the posterior capsule that occurs over time
* Occurs in approximately 35% of patients
* Yag laser capsulotomy can be used to fix situation --> opens up the posterior capsule thus allowing light to pass through (resulting in clear sight)

-----------------------------------------------------

159 - Which of the following nerves accompanies the profunda Brachii artery?

a. Ulnar
b. Musculocutaneous
c. Radial
d. Median

------------------------------------------------------

The correct answer is C

Profunda brachii artery

Parent Artery: Brachial
Branches and nerves run with:

1. Ascending branch: anastomoses with descending branch of posterior circumflex humeral of 3rd part of axillary artery. Participate in collateral circulation around shoulder joint
2. Descending branch: divides into middle and radial collateral arteries. Participate in collateral circulation around the elbow. Middle: descend behind lateral humeral epicondyle
3. Radial: descend in front lateral humeral epicondyle

Imp Relation: Passes laterally behind the humerus, associated with the radial nerve in the radial groove.

-------------------------------------------------------------

160 - The dicrotic notch on the aortic pressure curve is caused by

a. Closure of pulmonary valve
b. Rapid filling of left ventricle
c. Closure of aortic valve
d. Contraction of atria

--------------------------------------------------------

The correct answer is C

The notch indicates closure of the aortic valve, and comes at the end of diastole, as the pressure falls. The dicrotic notch represents the interruption of smooth flow due to the brief backflow of blood that closes the aortic semilunar valve when the ventricles relax.

----------------------------------------------------------------------

161 - The abductors of the vocal cords are

a. Transverse arytenoid
b. Oblique arytenoid
c. Lateral thyroarytenoid
d. Posterior Cricoarytenoid

--------------------------------------------------

The correct answer is D

The Laryngeal Musculature:

The intrinsic muscles of the larynx, all of which are innervated by the recurrent laryngeal nerve, include:

1) Posterior cricoarytenoid - - the ONLY abductor of the vocal folds. Functions to open the glottis by rotary motion on the arytenoid cartilages. Also tenses cords during phonation.

2) Lateral cricoarytenoid - - functions to close glottis by rotating arytenoids medially.

3) Transverse arytenoid - - only unpaired muscle of the larynx. Functions to approximate bodies of arytenoids closing posterior aspect of glottis.

4) Oblique arytenoid - - this muscle plus action of transverse arytenoid function to close laryngeal introitus during swallowing.

5) Thyroarytenoid - - very broad muscle, usually divided into three parts:

* Thyroarytenoideus internus (vocalis) - adductor and major tensor of free edge of vocal fold.
* Thyroarytenoideus externus - major adductor of vocal fold
* Thyroepiglotticus - shortens vocal ligaments


The cricothyroid muscle is considered to be an extrinsic muscle of the larynx because it is innervated by the external branch of the superior laryngeal nerve. It functions to increase tension in the vocal folds, especially at the upper range of pitch or loudness.

-------------------------------------------------------------------------

162 - The causative agent of Tinea Cruris is

A. Trichophyton concentricum
B. Cladosporum Werneckii
C. Mallassezia furfur
D. Epidermophyton floccosum

----------------------------------------------------------------------------

The correct answer is D


Tinea cruris is a 'herpes circinatus' that occurs in the fold of the groin. The microscopic picture is the same as in tinea corporis. The pathogens are almost exclusively three anthropophilic species, Trichophyton rubrum, Epidermophyton floccosum and Trichophyton mentagrophytes var. interdigitale, and are always found on the feet in these patients (tinea pedis). Erythrasma and intertrigo caused by yeasts must be included in the differential diagnosis.

-----------------------------------------------

163 - A patient with tubercular pleural effusion falls under Category _____ of WHO grading of TB

a. I
b. II
c. III
d. IV

-----------------------------------------------------------------


The correct answer is A

Type of patient Duration Regimen
CAT-I New sputum positive and seriously ill sputum negative pulmonary and extra pulmonary patients Intensive phase (2 months) INH+Ethambutol
+Rifampicin+Pyrazinamide
Continuation phase (4 months) INH+Rifampicin
CAT-II Relapse and treatment failures Intensive phase (3 months) 2 months INH+Rifampicin+
Pyrazinamide+Ethambutol
1 month
INH+Rifampicin+
Ethambutol+Pyrazinamide.
Continuation phase (5 months) INH+Rifampicin+
Ethambutol
CAT-III New smear negative(not seriously ill)pulmonary and extra pulmonary Intensive phase (2 months) INH+Rifampicin+
Pyrazinamide
Continuation phase (4 months) Rifampicin+INH


-----------------------------------------------------------------------

164 - Child changes a rattle from one hand to another at the age of

a. 3 months
b. 6 months
c. 9 months
d. 1 year

--------------------------------------------------------

The correct answer is B

What’s It Like To Be Six Months Old?


* I turn toward voices.
* I reach for toys and pick them up.
* I can hold an object in one hand and put it into the other hand.
* I briefly look for a dropped toy.
* I pick things up and I shake them.
* I turn objects upside down to get another view of them.
* I may roll over from my stomach to my back and from my back to my stomach.
* I play with my toes.
* I may help hold my bottle.
* I know my name.
* I may play games with people I know.
* I babble, squeal and repeat sounds.
* I sit by leaning forward on my hands. I can sit with support.
* I may be afraid of adults I don’t know.
* I know the faces of the people who are around me a lot.
* I may know what the tone of your voice means.

-------------------------------------------------------------------------

165 - Depolarizing neuromuscular blockade is associated with

a. Fasciculations
b. Fade on tetanic stimulation
c. Enhancement by volatile anesthetic agents
d. Antagonized by acetylcholine

-----------------------------------------------------------------------------------

The correct answer is A


The neuromuscular blockade by the classic pathway (depolarizing), begins when a drug binds to the a subunit of the nicotinic receptors like the molecule of AcetylCholinesterase does. In the beginning, an initial opening of the ion channel produces a contraction (fasciculation). After this, the depolarization of muscular membrane is sustained (persistent depolarization), since the drug is not broken by acetylcholinesterase, leading to neuromuscular block.

----------------------------------------------------------

166 - Thyroid storm after operation is due to

a. inadequate control of hyperthyroidism
b. massive bleeding
c. recurrent laryngeal nerve injury
d. postoperative infection

-----------------------------------------------

The correct answer is A


Thyroid storm is a rare and potentially fatal complication of hyperthyroidism. It typically occurs in patients with untreated or partially treated thyrotoxicosis who experience a precipitating event such as surgery, infection, or trauma.

Surgery - Now uncommon secondary to preventative measures, manipulation of the thyroid gland during thyroidectomy historically caused a flood of hormone release, often resulting in highly toxic blood levels. Exacerbations of thyrotoxicosis are still seen in patients taken too soon to surgery but are unusual in the antithyroid drug-controlled patient.

The decreased incidence of thyroid storm can be largely attributed. to the improved methods of diagnosis and therapy available today. In most cases, thyrotoxicosis is recognized before extreme debilitation occurs and is treated. by measures of predictable therapeutic value. Patients are routinely made euthyroid before surgery or treatment with 131-I. Under present-day therapy, using thiocarbamides, the glands have only minimal amounts of stored hormone, in contrast to the iodized gland facing the surgeons of six decades ago. Postoperative storm, formerly the most frequent kind of storm, has now been largely eliminated.

----------------------------------------------------------

167 - The commones type of Genital Tuberculosis is

a. Exosalpingitis
b. Endometritis
c. Interstitial Salpingitis
d. Exosalpingitis

---------------------------------------------

The correct answer is D

The tubes are involved in atleast 90% of cases following blood stream spread, probably the genital disease starts here. It begins in the submucosa at their outer ends and gradually progresses inwards, bombarding the endometrium with bacilli. The appearance of tuberculous tubes varies depending upon the type of spread. With hematogenous spread, the disease starts as endosalpingitis and direct spread from bowel or peritoneum is detected as exosalpingitis with tubercles on the surface and with surrounding adhesions.

-------------------------------------

168 - The parasympathetic secretomotr nerve supply to the nose is via

a. Anterior ethmoid nerve
b. Greater palatine nerve
c. Vidian nerve ( aldermans nerve )
d. Inferior orbital nerve

----------------------------------------------------------------------

The correct answer is C

In general, the nose is innervated by the olfactory nerve, branches of the ophthalmic and maxillary divisions of the trigeminal nerve, parasympathetic secretomotor fibers, and sympathetic fibers. The autonomic nerve supply to the nose regulates vascular tone, turbinate congestion, and nasal secretion. Parasympathetic fibers synapse in the sphenopalatine ganglion before innervating nasal mucosa. These fibers travel with the facial nerve, exiting at the geniculate ganglion as the greater superficial petrosal nerve. The vidian nerve is formed by the union of the greater superficial petrosal and deep petrosal nerves and carries the fibers to the sphenopalatine ganglion, where they synapse.

-------------------------------------------------------------------------

169 - A patient who is using a direct-acting sympathomimetic agent as a decongestant is most likely to be taking

a. Ephedrine
b. Amphetamine
c. Phenylephrine
d. Metaraminol

-------------------------------------------------------------------------

The correct answer is C


Phenylephrine is sympathomimetic vasoconstrictor that has been used as a nasal
decongestant for many years. It is a relatively selective alpha-adrenoceptor agonist. The majority of the sympathomimetic action is due to direct stimulation of the
adrenoceptors and relatively little is due to an indirect effect via release of
noradrenaline.

-----------------------------------------------------------------------------------

170 - The fourth heart sound is caused by

a. Closure of aortic and pulmonary valves
b. Vibrations in the ventricular wall during systole
c. Ventricular filling
d. Closure of the mitral and tricuspid valves


----------------------------------------------------------------------------

The correct answer is C

The fourth heart sound is due to atrial contraction inducing ventricular filling towards the end of diastole. They are never audible in normal subjects. A fourth heart sound is the result of powerful atrial contraction filling an abnormally stiff ventricle.

------------------------------------------------------------------------------

171 - Power grip of the hand depends on

a. Short flexor of fingers
b, Lumbricals of hand
c. Long flexors of fingers
d. Palmaris Brevis

----------------------------------------------------------------------------

The correct answer is C

As the hand approaches the objects to be picked up, the extensor tendons posture the hand and fingers in preparation for the grip. The long flexors then clamp the fingers against the palm with the thumb usually in adduction.

The strength of this power grip in proportional to the range of flexion in the smallest joints, so that any cause of interphalangeal joint stiffness leads to serious weakening of the power grip. The final torque of this power grip in grasping objects comes from thenar and hypothenar intrinsics and the loss of this transverse arch movement is noted in ulnar nerve palsy.

The extensor digitorum and forearm flexor digitorum superficialis muscles play an important role in power force matching.

----------------------------------------------------------------------------------------

172 - Popcorn type of Reed Sternberg cell is seen in the following type of Hodgkin's lymphoma

a. Lymphocyte-Rich
b. Mixed cellularity
c. Lymphocyte predominant
d. Lymphocyte depletion

--------------------------------------------------------------------------------------------

The correct answer is C

The lymphocyte-predominant type differs from classical Hodgkin disease with respect to the background and the presence of “popcorn” or lymphocytic and histiocytic–type (L&H) cells. Popcorn or L&H cells express CD45 and frequently CD20 but not CD15, CD30, or vimentin. The Reed-Sternberg cell and its variants are not seen in this type of Hodgkin disease.

-----------------------------------------------------------------------------------

173 - Antimicrobials that produce hemolysis in G6PD Deficiency

a. Ciproflxacin
b. Nitrofurantoin
c. Norfloxacin
d. Mandelamine

----------------------------------------------------

The correct answer is B


Glucose-6 phosphate dehydrogenase deficiency (G6PD) is a hereditary enzyme defect that can result in anemia.People with this deficiency are not normally anemic. Instead, they may have occasional episodes when their red blood cells break down and stop functioning.

Drugs that can precipitate this reaction include:

* Anti-malaria medications
* sulfonamides (antibiotic)
* nitrofurantoin
* others

----------------------------------------------------------------

174 - The predominanat constituent of the pale yellow gallstones in the gall bladder is:

a. Mucin Glycoprotein
b. Calcium Carbonate
c. Cholesterol
d. Calcium phosphate

--------------------------------------------------------

The Correct Answer is C

Pure cholesterol stones (uncommon) are pale yellow and have a glistening crystalline center with a radiating appearance.

Mixed cholesterol stones are the commonest form and because of varying proportions of calcium salts and bilirubin, these stones have a lamellated or layered structure and variegated appearance with color mixtures ranging from gray-white to yellow to black flecked surfaces and often black centers.They account for about 80 percent of gallstones.

Pigment stones are small, dark stones made of bilirubin.

------------------------------------------------------------

175 -Most common drug resistance in tuberculosis, in India, is found for

a) INH
b) Rifampicin
c) Streptomycin
d) Ethambutol

-----------------------------------------------------------

The correct answer is A

The Indian Council of Medical Research (ICMR) undertook drug resistance studies during 1965-67 in nine urban areas of the country. The first study was on patients who had denied any history of previous treatment, while in the second study, patients with and without previous chemotherapy were included. The results showed that in the first study resistance to isoniazid ranged from 11-20 per cent, to streptomycin from 8-20 percent and to both drugs from 4-11 per cent. The second study showed resistance to isoniazid to range from 15-69 percent, to streptomycin from 12-63 per cent and to both drugs from 5-58 per cent.A decade later, a study at the Government Chest Institute and Chest Clinic of Government Stanley Hospital (GCI-SH), Chennaai20 yielded results similar to those in earlier ICMR surveys, indicating that the prevalence of initial drug resistance had not risen during the span often years.

However, both the above studies were undertaken in the pre-rifampicin era and are not of relevance in the present setting.

In the early 1990s, a retrospective study done at New Delhi showed a high level of initial drug resistance to isoniazid (18.5%) and a low level of resistance to rifampicin.

At present resistance to Isoniazid is highest when compared to other anti- tubercular drugs.

--------------------------------------------------------

176 - Chvostek's sign is acontraction of facial muscles and upper lip when the examiner taps the facial nerve in front of the ear. A positive sign suggests which of the following conditions?

A. Hypercalcemia
B. Hypocalcemia
C. Hyperkalemia
D. Acidosis

----------------------------------------------------------

The correct answer is B

Chvostek's Sign: Hypocalcaemia is associated with increased neural excitability, which may be demonstrated by gentle percussion over the proximal part of the facial nerve (as it exits from the parotid gland). The test is positive if this maneuver evokes involuntary facial muscular twitching.

------------------------------------------------------------------

177 - 90% of brain growth is achieved by the

a) 2nd year
b) 3rd year
c) 5th year
d) 15th year

--------------------------------------------------------------

The correct answer is C

BRAIN GROWTH:

An adult's brain weighs approximately 3 pounds. The brain increases its weight more than three times between birth and adulthood.

* Birth: The brain weighs approximately 350 grams.
* 6 Months: The brain is at approximately 50% of its adult weight.
* 1 Year: The brain weighs approximately 1400 grams. This is approximately 50 - 60% of its adult weight.
* 2 Years: The brain is at 75% of its full weight.
* At age 3, a child’s brain has reached about 90% of its full potential.
* 5 Years: The brain is within 90% of its adult weight.
* 10 Years: The brain is within 95% of its ultimate weight.
* 12 Years: Full brain weight of 3 pounds.

-------------------------------------------------------------------------

178 - Which is the commonest site for extra-pulmonary TB?

A. Pleura
B. Lymph Node
C. Intestine
D. Meninges

---------------------------------------------------------------------------------

The correct answer is B

Tuberculosis may affect any organ in the body. Extra-pulmonary TB may account for 5-30% of TB cases. In order of frequency, the extra pulmonary sites most commonly involved in tuberculosis are the lymph nodes, pleura, genitourinary tract, bones and joints, meninges and peritoneum. TB disease may also manifest as disseminated tuberculosis or miliary tuberculosis.

-------------------------------------------------------------------------------------

179 - Podophyllin is used in the treatment of

a) Genital Warts
b) Common warts
c) Molluscum contagiosum
d) Herpes simplex

-------------------------------------------------------

The correct answer is A


Condyloma acuminata, (venereal warts) are caused by a virus known as "Human Papilloma Virus" (HPV).

There are two categories of warts, clinical and subclinical. Clinical warts appear as tiny, cauliflower-like, raised lesions around the opening of the vagina or inside the vagina. These lesions appear flesh-colored or white, are not tender and have a firm to hard consistency. If they are on the outside of the vagina or vulva, they are generally symptomatic, causing itching, burning, and an uncomfortable sensation during intercourse. If they are inside the vagina, they generally cause no symptoms.

Podophyllum resin can be applied directly to the wart, followed by washing off the residual podophyllin in 3-6 hours. This effective approach runs the risk of podophyllin toxicity. This is a minor issue if the wart is very small and you use tiny quantities of podophyllin. If you use large amounts, or apply it inside the vagina, toxicity is a real issue.

---------------------------------------------------------

180 - The Incubation period of Leprosy is

a) 2-4 days
b) 2-4 weeks
c) 2-4 months
d) 2-4 years

--------------------------------------------------------

The correct answer is D

Summary of Leprosy

* Infection with Mycobacterium leprae
* Bacteria multiply in the macrophages and Schwann cells of peripheral nerves
* Clinical spectrum: from tuberculoid (paucibacillary) to lepromatous (multibacillary)
* Thickened nerves with neuritis: trophic, motor and sensory disturbances
* Neuropathy leads to paralysis, trophic ulcers, blindness, mutilations
* Skin : numb white area with elevated edge (TT) to diffuse infiltration with nodules (LL).
* In lepromatous leprosy also involvement of deeper tissues (testes, tongue, eyes, etc.)
* Diagnosis clinical, Ziehl staining of smears (skin lesion, nose, earlobe)
* Treatment of leprosy with dapsone, rifampicin, clofazimine
* Leprosy reactions: type 1 (change in immunologic defence) and type 2 (immune complex)
* The majority of infections do not give rise to symptoms (only to a positive lepromine test). After infection there is an incubation period of 2-15 years (the mycobacteria multiply slowly).

------------------------------------------------------

No comments: